Preventive Medicine 1

September 28, 2017 | Author: Ong Kie | Category: Cohort Study, Preventive Healthcare, Health Care, Public Health, Statistical Hypothesis Testing
Share Embed Donate


Short Description

Prev Med Review questions...

Description

PREVENTIVE, FAMILY & COMMUNITY MEDICINE EPIDEMIOLOGY, HEALTH STATISTICS & RESEARCH . 1. The 95 percent confidence interval for a sample mean value represents: A. The range in which the true population mean is most likely to exist. B. The author‟s belief that the presented data are true. C. The range in which 95 percent of the sample values fall. D. A test for statistical significance of the mean value. 2.

OF the following statements, which best describes the specificity of a clinical test? A. The proportion of non-diseased individuals who have a negative test B. The proportion of diseased individuals who have a positive test C. The proportion of diseased and non-diseased individuals who are correctly categorized by the test D. The proportion of test-negative individuals who are free of disease

Data from clinical studies are reported frequently in the form of “2 by 2” tables. A 2 by 2 table for a hypothetical test is shown below: Test Result Disease Present Disease Absent Positive 50 20 Negative 30 50 3.

4.

5.

6.

7.

8.

9.

What is the test sensitivity? A. 63 percent B. 29 percent

C. 71 percent D. 38 percent

What is the positive predictive value? A. 71 percent B. 47 percent

C. 38 percent D. 33 percent

You have decided to conduct a research project in your office. A number of steps should be taken to assure a successful study. What is the first step you should complete in planning your project? A. Formulate the study question. B. Consult with a statistician. C. Design a form for collecting data. D. Determine the necessary sample size. You have conducted a clinical trial in which you measured blood pressure in the same patients during a control period and after 2 weeks of treatment with an experimental antihypertensive medication. Systolic blood pressure was the characteristic you measured. You now want to determine whether there was a significant difference in the average (mean) systolic blood pressure between the control and treatment periods. Which of the following is the most appropriate statistical procedure for this determination? A. Paired t test C. two sample t test B. Chi-square D. correlation coefficient Experimental studies: A. Are the most effective studies to assess causal relationships. B. Are synonymous with observational studies. C. Require that subjects be randomly assigned to exposure and nonexposure groups at all times. D. Excludes community trials. Which of the following statements is true regarding prospective cohort studies? A. They measure the relative risk of developing a disease for an individual with an exposure. B. Individuals are grouped on the basis of their disease status. C. Subject attrition is not a major potential flaw. D. They cannot be used to observe multiple exposures and conditions. E. They are poor in elucidating temporal relationships between exposure and disease. Which of the following statements is true regarding cross-sectional studies? A. They assess exposure and disease status simultaneously. B. They can yield information regarding disease incidence. C. They are more time-consuming to perform than case-control or cohort studies. D. They are an effective design to determine casual relationships. E. They are not an appropriate methodology to draw conclusions from chart databases or census data.

10. Which of the following is an example of cohort studies? A. Men are grouped as to whether or not they have had radioactive exposure; then their existing medical records are evaluated for a diagnosis of lung cancer. B. Women are grouped as to whether or not they have chronic obstructive lung disease; then they are asked about their occupational history. C. Women are given a questionnaire assessing weight and menstrual symptoms.

1

D.

A group of elderly veterans are followed for adverse reactions to the flu vaccine.

11. A family physician believes that sterile butterfly stripping is just as effective as suturing with regard to the closure of simple lacerations. To test his theory, he uses sterile butterfly strips to close the next 25 lacerations that he sees in his office. He reports that 96 percent of the lacerations had excellent results. Which of the following statements is true regarding this experiment? A. This is a case-series design. B. This design is an ineffective one for the initial study of the phenomenon. C. It can be assumed that all lacerations were equivalent with regard to location, extent, and mechanism of injury. D. The next phase of study should be a prevalence design. E. Sterile butterfly stripping is at least equivalent to suturing with regard to its ability to attain adequate wound closure. 12. In double-blind randomized clinical trials: A. The distribution of characteristics between the exposure and nonexposure groups is determined by chance. B. The investigator is aware of who is and who is not getting the exposure. C. The subjects are not aware that they are taking part in a research protocol. D. The subjects are permitted to choose whether they want to be in the exposure or nonexposure group. OCCUPATIONAL HEALTH, INDUSTRIAL MEDICINE & MEDICAL SOCIOLOGY 13. The following agent/s causing diffuse mottling of the lungs: A. Bagasse C. Silica B. Candida D. TB 14. Substitute/s causing neurological problems: A. Mercury B. Silica 15. Pneumoconiosis is caused by: A. Asbestos B. Cotton dust

C. Asbestos D. Bagasse C. Bagassosis D. Bagasse

16. This mineral dust is incurable and may lead to the development of Tuberculosis: A. Silica C. Cryptococcus B. Histoplasma D. Aspergillosis 17. An important component of an occupational history is: A. temporal relationship of symptoms with work time B. associated symptoms among family members C. family illnesses D. dietary history 18. The most exposed industry to ergonomic problems: A. Manufacturing C. Fishing B. Mining D. Agriculture 19. Threshold limit value – is the time weighted average concentration which nearly all workers can be exposed daily without adverse effect for the following number of work hours/workweek: A. 40-hour workweek C. 48-hour workweek B. 12-hour workday D. 6 hours work shift SANITATION, ENVIRONMENTAL MEDICINE & HUMAN ECOLOGY 20. Considered most dangerous with regards to possible disease spread: A. clinical carrier C. non-immunized B. clinical stage patient D. terminally ill patient 21. This allows organic material to undergo bio and photo degradation, resulting in simple organic molecules that can actually be beneficial to the environment: A. Composting C. Source reduction B. Reuse and recycling D. All of the above 22. An example of an agent-environment interaction is: A. Population of a “carrier” making use of an infected water supply both for drinking and food preparation B. Vitamin C tablets placed in colored bottles C. Fly control program D. Business‟s favorite of eating “sushi” in Japanese restaurants

2

23.

What treatment is needed if the bacteriological quality of water is 5,000A. Double treatment C. Disinfection B. Look for another source D. Chlorination

50,000 coliforms/100 ml?

24.

The most critical step in waste reduction and recycling is: A. Waste segregation C. collection of solid waste B. incineration D. transfer and transport

25.

The main objective of solid waste processing is: A. to recover whatever may be reusable B. to provide livelihood for scavengers C. to discover other sources of income D. to minimize bulk of waste to be dumped at sea

26.

The most important public health issues addressed by correct waste management: A. Composting of solid waste from laboratories B. Control of vectors of infectious disease C. Eradication of mosquito causing Dengue DC. Improvement of parks and public places

FAMILY MEDICINE 27.

Biopsychosocial Model focuses on: A. Multifactorial causes of illness B. Disease oriented

28. The A. B. C. D.

C. Organ system dysfunction D. Biological orientation

most challenging and rewarding stage for the physician in the Family Illness Trajectory: Major therapeutic efforts Accuracy of diagnosis Early adjustment to possible outcome Adjustment to permanency of outcome

29. It is process which encompasses screening for abnormalities, early detection of disorders that can be alleviated, and likewise the prevention of ill-health: A. Family Health Care C. Family Illness Trajectory B. Family Life Cycle D. Family Assessment Tools 30. The nature of illness whose characteristics of experience provide little time for physical and psychological adjustment for family members: A. Acute illness C. Terminal illness B. Chronic illness D. Debilitating illness 31. The most difficult stage of the Family Illness Trajectory for the patient: A. Accuracy of diagnosis B. Adjustment to permanency of outcome C. Major therapeutic efforts D. Early adjustment to possible outcome 32. This household is in the typical case-economically independent subsisting in the first instance from the occupational earnings of the husband-father: A. Nuclear family C. Extended family B. Corporate family D. Blended family 33. An acronym that stands for factors affecting health which can be considered resource and pathology: A. SCREEM C. Family APGAR B. Family Circle D. Family Genogram 34. Component of the Family Genogram that projects the dynamism of family relationships: A. Functional chart C. Family Illness and history B. Family resources D. Pedigree or Family Tree 35. A Genogram using functional symbols: A. depicts relationships B. focuses on the most ill family member C. emphasizes the problem of the patient D. therapeutic interventions 36. The “therapeutic Triangle” in Medicine refers to which of the following? A. Patient/family/health care team B. Patient/family, health care institution, health care team

3

C. Patient/family, friends/extended family, health care team D. Patient, health care team, community 37. What stage of the Family Life Cycle is characterized by parents developing adult type relationships between the grown children and themselves? A. Family with adolescent C. Unattached Young Adult B. Newly married couple D. Family in later life 38. What family process involves ordered sequences of interaction that typify how family functions? A. Family pattern C. Triangulation B. Disengagement D. Enmeshment 39. Which of the following is characteristic of the hospice program? A. available 24 hours a day B. designed for marginalized population C. renders only medical care D. composed of nurse and physician only 40. During the interview process, which of the following will most likely put the patient on the defensive, that is, make the patient feel that his or her problem or behavior must be justified and defended? A. Beginning the question with “Why?” C. Laundry-list questions B. Direct question D. Open-ended questions 41. This is a component of the family health care plan which involves health maintenance for each Family member: A. Preventive C. Therapeutic B. Diagnostic D. Rehabilitative 42.

A family meeting should be done in the following situation: A. 16 y/o old female, first pregnancy B. 12 y/o female with acne C. 4 y/o old boy with acute nasopharyngitis D. 30 y/o old male for annual employment physical

HEALTH ADMINISTRATION & MANAGEMENT, PUBLIC HEALTH 43. A A. B. C. E.

characteristic of Primary health care that differentiates it from the traditional health delivery system: Community participation in the stages of planning Assurance of financial backing of the government Coverage of the majority of the population Drug provision for all indigent patients

44. Fecal occult blood is a screening test for: A. colorectal CA B. cervical cancer

C. Inflammatory bowel disease D. hemorrhoids

45. Criteria that justifies doing screening test: A. Can identify patients likely to have the disease at pre-symptomatic stage B. Can identify patients who have the disease at symptomatic stage C. Can identify patients who will not benefit from treatment D. Can identify patients who will not benefit from rehabilitation 46. Which of the health maintenance plan is true for adult patients? A. FBS should be done for obese 40 y/o female with family history of hypertension B. Fecal occult blood is recommended for 50 years old and above C. VDRL/RPR should be done on all males D. Electrocardiogram should be done on all 50 years old and above 47. Which of the following statements about chemoprophylaxis is true? A. Low dose aspirin for men 65 years old and above prevents heart disease B. Estrogen with progestin increases the risk of endometrial cancer C. Estrogen replacement therapy decrease the risk of osteoporosis when started in women 40 years old and below D. Chemoprophylaxis is warranted only in younger patients 48.

A A. B. C. D.

health promotion measure: Utilization of health/medical services Use of mosquito repellant Vitamin supplementation to increase body resistance Isolating patients with infectious diseases

4

49. A rehabilitation measure: A. Organized feeding program B. Use of iodized salt because of prevalence of goiter C. Teaching to deaf mute children sign language or/and lip reading D. Prescribing eyeglasses to those with initial stage of nearsightedness (myopia) 50. The process of isolating patients with Flu falls under: A. Specific protection C. Disability limitation B. Early diagnosis and treatment D. Health promotion 51. Factors affecting community reaction: A. Previous exposure B. Chance contact

C. Virulence of agent D. Volume of agent

52. An epidemic occurs due to: A. Absence of an immune barrier B. Inadequacy of therapeutic measures C. Emergence of a new strain of the disease agent D. Migration of a population group 53.

Considered a missed human reservoir of disease: A. Undiagnosed and therefore unreported case of a disease B. Patient wrongly diagnosed because of inadequate laboratory work-up C. Seek medical consultation D. Correctly diagnosed patient but unreported case

54.

Increase in life expectancy is mainly due to: A. Decrease in mortality in the younger age groups B. Improvement in health information dissemination C. Better diagnostic facilities D. Discovery of effective drugs

55.

Preventive measures are most effective when applied to a person: A. unaffected C. affected symptomatic B. affected asymptomatic D. without complications

56.

Mila‟s father and mother both have diabetes. She consulted because of health is explored in the history? A. Genetic inheritance C. Social factors B. Belief systems D. Nutrition

polyuria.

Which determinant of

57. Denotes a non-human carrier of infection‟s organisms that can transmit disease directly to humans: A. Vectors C. Plasmodium Sp. B. Rodents D. All of the above 58. Step A. B. C. D.

in epidemiological method of investigation include: Discovering historical occurrence of diseases Formulation of community diagnosis of health and disease Estimation of morbidity and mortality rates Making critical appraisal of existing information

59.

Which statement is TRUE about the Control of Diarrheal disease program? A. Emphasizes home therapy by continued feeding and increase fluid intake B. Aims to eradicate food and water borne diseases C. Targets children under 5 - 7 years old D. Promotes the use of anti-diarrheal drugs as mainstay of treatment

60.

In making objectives for health education, the following is important: A. They are worded in terms of learner behavior B. An affective objective requires hands-on experience C. Evaluation clarifies what need to be strengthened D. Instructor ability is important in planning the learning activity.

61.

Partnership approach to community health development is among the: A. private sector, government sector and the community B. community and its local government leaders C. international and national agencies D. private and government practitioners

62.

The first contact of the community to the health chain, as defined by the

PHC system is the:

5

A. village health workers B. intermediate level health workers

C. hospital personnel D. barangay captain

63.

The A. B. C. D.

health status of a community may be measured or expressed in terms of: its statistical indices of morbidity and mortality availability of health care services availability of public utilities utilization of health care services

64.

Community planning should start with: A. formation of objectives B. identification of roles and their relationships C. educational analysis D. determination of resources

65.

Identifying and prioritizing health problems is part of this planning process: A. situational analysis C. plan investigation B. plan implementation D. plan formulation

66.

Best people to identify and prioritize community health problems: A. community residents C. health workers B. social workers D. consultants

67.

The initial step in planning for the provision of health and medical care A. define the health problems C. set the objectives B. determine the projected budge

services for a community is to:

D. decide on what services to provide

68.

The gross domestic product adjusted with the net factor income from the A. Gross National Product C. Gross Domestic Product B. Gross Value Added D. Consumer Price Index

rest of the world:

69.

The objectives in the tertiary prevention of non-communicable diseases A. help the patient function maximally within the restrictions imposed by the disease B. removal of the agent from the environment C. screening D. immunization

include the following:

70.

In computing the cost of illness, the following data are needed: A. Average daily income or wage B. Average days of non-morbid condition C. Average cost of savings D. Number of death

71.

Health education involves: A. process of translating knowledge into action B. telling people what to do C. giving lectures about health D. issuing rules and regulations to stop a bad habit

72.

The primary objective of health education is to: A. improve health practices B. transfer technology to lay personnel C. impart knowledge D. promote health in general

73.

Public health services (national and local) must give emphasis on: A. prevention, promotion and maintenance of health B. treatment of diseases C. rehabilitation of the disabled D. provision of emergency and first aid services

74.

The role of government agencies in solving community health problems: A. guide, assist and facilitate community efforts in meeting its needs B identify the problem of the community C. provide funds for the project D. provide comparative statistics

75.

In public health, the most frequent problem of scarcity of resources is A allocate resources according to needs B. transfer of funds

best met by:

6

C. complete for national funds D. increase manpower training 76.

The managerial approach to motivation in health development is: A. to develop a shared responsibility for achieving organizational and individual goals by contributing on the basis of his interest and abilities B. manipulate workers by considerate treatment C. induce workers to perform by giving them high wages D. motivate workers to do overtime work

77.

The A. B. C. D.

78.

Evaluation of a health program is necessary in order to: A. determine if other objectives have been attained B. prematurely stop the program implementation C. allocate available resources to another program D. re-assign health manpower to other worthy programs

79.

The basic indicator most often used to reflect the economic health of a A. infant mortality rate C. migration rate B. infant mortality rate D. population density

most effective change in health behavior occurs when: the individual incorporates the change into his system of values the educator is an attractive figure a reward is offered if the change is adopted the individual is threatened with punishment if he does not accept the Change

country is:

HEALTH CARE: PHILIPPINE SETTING 80.

The most common type of attendant at birth utilized by Filipino women: A. Hilots C. physicians B. Midwives D. nurses

81.

Maternal health status of the Philippines is still poor because of the A. doctors attend to ¼ of deliveries B. birth rate is relatively low C. maternal death rate is relatively low D. ¾ deliveries are in hospital

82.

In the Philippines, the primary factor to consider in the design of the health care delivery system is: A. prevalent diseases C. education of the masses B. population growth D. nutritional requirement

83.

Republic Act 7875 is also known as “The National Insurance Act of 1995” covers: A. all Filipino citizens C. government employees only B. private employees only D. unemployed citizens

84.

The A. B. C. D.

85.

This specifically consists of a core list of drugs approved and authorized A. Essential Drug List C. Positive Drug List B. Generic List D. Prohibited Drug List

by the DOH:

86.

By tailored procurement of drug by government, it will make available to A. the best drug at least cost C. bulk buying B. the expensive drugs at low cost D. the cheapest drug

its own clientele:

87.

DOH Office that is responsible for handling matters pertaining to drugs, licensing is: A. Standards and regulations C. Hospital services B. Management services D. Public Health services

88.

following:

priority or target group in the National Anti-TB Program as recognized sputum positive cases sputum negative but with history suggestive of TB sputum negative cases but with possible X-ray findings sputum negative but with clinical findings suggestive of TB

by PHILHEALTH and DOH is:

laboratories, facility and professional

DOH‟s goal of improving the general health status of the population require: A. Reduce infant/child mortality rate C. Increase total fertility rate B. Decrease life expectancy D. Increase growth rate

7

89.

PHILHEALTH Benefits Package includes: A. Normal Spontaneous Delivery of first two (2) births B. Outpatient psychotherapy and counseling for mental disorder C. Drugs and alcohol abuse and dependency treatment D. Home and rehabilitation services

90.

The primary aim of decentralization is: A. To increase resource base for primary care, shifting from central to peripheral B. To decrease health center utilization rate C. To give accountability to political authority D. To relegate procurement of drugs and supplies to LGUs

91.

Prevention of cancer recurrence and complications falls under what level Control Program: A. Tertiary C. Primary B. Secondary D. Primordial

92.

True A. B. C. D.

93.

The A. B. C. D.

of prevention of the DOH Cancer

of TB in the Philippines: DOTS aims to improve treatment compliance More common in age group 60 and over Three times more common among females than males Remains the # 1 leading cause of mortality in the country most effective control measure for Tuberculosis which is prevalent in the Philippines is: Mass BCG vaccination “Clean Air” Campaign to eliminate pollution Massive nutritional correction and support Promotion of chemoprophylaxis for contacts/exposed population

94.

Which of following tops the list of major causes of intestinal parasitism in the Philippines: A. Ascaris Lumbricoides C. Capillaria Philippinensis B. Hookworms D. Trichuris Trichiura

95.

Which of the following has the highest prevalence of all the risk factors to cardiovascular diseases according to the DOH Cardiovascular Protection Program? A. hypertension C. obesity B . smoking D. stress D. Bereavement must cater to the family members also

96.

In 2003, National Nutrition Survey (FNRI) showed that the prevalence of this vitamin and mineral deficiency remains high for: A. Vitamin A & iodine C. Vitamin A & Zinc B. Vitamin B & iron D. Vitamin D & iodine

SIMPLE MULTIPLE CHOICE. CHOOSE THE BEST ANSWER. GOVERNMENT PROGRAMS 97. If both parent are Medicare members, the dependent children may claim benefit from membership of: a. Mother c. Father b. Both mother and father d. One can choose 98. Family planning means: a. Voluntary descisions and positive action of couples to have a desired number of children b. Intra-uterine device and pills c. Legal abortions d. Knowledge of the different family planning methods 99. The major activities undertaken in the maternal and child health program does NOT include

a. b. c. d. 100. a. b. c. d.

Immunization of pregnant mothers against measles Proper pneumonia prevention Promotion of prenatal care Monitoring of growth and health status of infants and children

In the “Under Five” Program, the best screening procedure for tuberculosis: Sputum examination Surveillance Tuberculin testing Chest x-ray

8

101. A. B. C. D. 102. is:

Drugs not covered by patent protection and which are labeled solely by their international proprietary name A. B. C. D.

103. A. B. C. D.

The prerequisite for any rational drug policy is Availability Accessibility Quality assurance Affordability

Complementary drugs Essential drugs Generic drugs Core drugs

PhilHealth gives medical coverage to dependents of members who are: parents over 60, children below 21 parents over 65, children below 21 parents over 60, children below 18 parents over 65, children of any age

104. Philhealth benefits vary according to the following except: A. Relative unit value of surgery C. Type of hospital B. Actual cost of care D. Type of physician 105. Phil-Health sets standards, guidelines and procedures prior to allowing doctors and hospitals to become part of the NHIP. This process is called: A. evaluation C. recognition B. accreditation D. acceleration 106. a. b. c. d.

The Family Planning program consist of the foloowing EXCEPT: Promotion of knowledge on the use of contraceptive devices Helping childless couples to bear children Encouraging legal abortion Providing family planning services

a. b. c. d.

The following preventive measures are included in the National TB Control Prgram EXCEPT: Giving chemoprophylaxis for those not yet infected BCG vaccination of eligible population Early treatment of infective TB cases Health education

a. b.

According to the revised Expanded Program of Immunization, the contraindication/s to immunization is/are: Malnourished child c. Diarrhea Temperature of 37.5 – 38o d. Convulsions after DPT

a. b. c. d.

A greater measure of success of a TB control program could be expected if activities were centered on: Wider coverage of BCG immunization Development of more effective treatment methods Amelloration of the socio-economic status of the general population Identification and treatment of early cases

a. b.

Toxic effects of INH has been reported as: Diarrhea Hepatitis

a. b.

In an “under six” program the best screening procedure for tuberculosis is: Chest x-ray c. Tuberculin test Sputum examination for AFB d. B and C only

a. b.

All are ingredients of Oresol (oral rehydration fluid) EXCEPT: Calcium chloride c. Potassium chloride Sodium chloride d. Glucose

a. b.

Routine immunization of children include the following EXCEPT: BCG c. Measles Thyphoid d. Diphtheria

a. b. c.

The folowing immunizations are recommended for 3-months old children consulting at an “under six clinic”: DPT d. A and B Measles vaccine e. A and C BCG

107.

108.

109.

110.

111.

112.

113.

114.

c. Chorioretinitis d. Extrapyramidal symptoms

9

115. a. b.

Immunization is contraindicated among Malnourished children c. Children with diarrhea Breastfed babies d. Children with prolonged febrile illness

a. b. c. d.

Contraindications for administering a live attenuated vaccine include all of the following EXCEPT: Acute febrile illness Recent administration of immune globulin host Immunosuppressive disorder or compromise Administration of another live vaccine

a. b.

BCG vaccination is administered on the right deltoid are a by: Intramuscular method c. Intradermal method Subcutaneous method d. Scratch method

a. b.

The recommended WHO schedule for measles vaccination is at: Birth c. 3 months 6 weeks d. 9months

a. b.

Maternal antibodies convey passive measles immunity to infants probably until age: 12 to 15 months c. 3 to 6 months 6 to 9 months d. 18 – 24 months

a. b. c. d.

The The The The The

a. b. c. d.

The following are the statements of Policy of the Generic Act of 1988, EXCEPT: Promote, encourage, and require the use of generic terminology Penalize any violation of the Act Emphasize the scientific basis for the use of drugs Ensure adequate supply of drugs with generic names

116.

117.

118.

119.

120.

121.

descision to give the rabies vaccine is influenced by any of the following EXCEPT: severity of the bite condition of the animal at the time of exposure part of the body that was bitten condition of the animal 15 days after axposure

122. The following organizations jointly with the Department of Health are involved in the education drive for the Generics Acts, EXCEPT: a. Department of Finance b. Philippine Information Agency c. Department of Local Governments d. Department of Education, Culture & Sports 123. a. b. c. d.

Which of the folowing would be best reflect the inadequancy of maternity care services in the community? Small proprotion of hospital births Small proportion of deliveries attended by physicians Large proprotion of maternal deaths due to hemorrhage None of the above

a. b.

The leading cause of maternal death in the Philippines is: Abortion c. Toxemia of pregnancy Infection d. Hemorrhage

a. b. c. d.

High risk in maternal mortality are mothers Under 15 years or above 45 years 17 – 20 years 20 – 25 years 30 – 35 yers

a. b.

The most effective method of contraception is: IUD c. Rhythm method Pills d. Barrier or condoms

a. b. c. d.

This is NOT one of the priority needs of a mother Nutrition promotion and growth surveillance Control of infectious disease including tetanus Maternal care Nutrition supplementation

a. b.

The target group for tetanus toxoid immunization is: Women 12-59 years c. All persons Women 15-44 years d. All women regardless of age

124.

125.

126.

127.

128.

10

129. a. b.

At birth a Filipino baby usually weighs: 2000 gms 2500 gms

a. b. c. d.

Prevention of prematurity can be done by: Promotion of preconceptional care Improvement of obstetrical service Improvement of maternal nutrition Improvement of prenatal care

a. b. c. d.

Among the following factors, which has the LEAST influence on the many infants during the first day of life? Poor environmental sanitation Incompetent attendance at delivery Inadequate prenatal care Maternal malnutrition

a. b.

Chemotherapeutic agents are applied to the eyes of the newborn to prevent Juvenile cataract c. Syphilis Gonorrhea d. Trachoma

130.

131.

132.

PUBLIC HEALTH 133. a. b. 134. a. b.

c. 3000 gms d. 4000 gms

Long-standing lack of vitamin A is maifested in the form of this ocular lesion: Keratomalacia c. Nyctalopia Corneal xerosis d. Bitot‟s spots If a 6-month old baby is suffering from diarrhea, the following should be told to the mother: Stop breast-feeding c. Give “am” and tea only Stop the usual food given d. None of these

PUBLIC ADMINISTRATION 135. The role of governmental agencies in solving community health problems is to: a. Identify the problem of the community b. Provide funds for the project c. Provide comparative statistics d. Guide, assist, and facilitates community efforts in meeting its needs 136. a. b.

In public health, the most frequent problem is scarcity of resources. This can be best met by: Transfer of funds c. Allocate resources according to needs Compete for national funds d. Increase manpower training

EPIDEMIOLOGY 137. In writing a research proposal, the formal statement about the most likely outcome of the proposed research falls under: a. Literature review c. Hypothesis b. Methodology d. Objectives 138. a. b.

The factors that effect the reliability of an instrument are the following EXCEPT: Observer variation c. Lack of congruency Lack of precision d. Lack of validity

a. b.

A control or comparison group is necessary in: Descriptive studies c. Cross-sectional studies Prevalence studies d. Prospective studies

a. b. c. d.

The importance of reviewing the available literature related to research problem is To prevent duplication of work that has been done before It may lead to refine the problem definition To find out the authors of these works To provide good reasons for others to support the proposed research

139.

140.

141. Among the following demographic characteristics, which does not contribute to high fertility in the Philippines? a. High marriage rate c. Increasing life span b. Young population d. Low median age on marriage 142. a. b. 143.

The greatest contributor to population growth in ASIA today is: Increased mortality rate c. Decreased birth rate Increased birth rate d. Decreased mortality rate Life expectancy is lower among the

11

a. b.

Married Single

a. b. c. d.

In the choice of a problem for research, which of the following criteria is the LEAST important? Availability of technical and logistic support Probability of solving the problem by research Probability of acceptance for publication Useful applicability of the results

a. b.

Because of its inherent weakness, this method is not utilized in the collection of scientific data: Census c. Registration method Questionnaire method d. Case record method

144.

145.

146. studied a. b. 147.

c. Young professionals d. Separated / divorced

The following epidemiologic approach may be utilized when the disease being investigated has not been yet in the past: Descriptive c. Cohort prospective Retrospective d. Experimental

a. b.

This type of study can best demonstrate a cause and effect relationship: Experimental study c. Analytic study Descriptive study d. Cross-sectional

a. b.

The possibility of cause-effect relationship is difficult to assess in: Experimental c. Cohort Cross-sectional d. Case-control

148.

149. To determine if there is any relationship between oral contraceptive use and chlamydia infection, an investigator selected a sample of 100 women attending a social hygiene clinic. He then measured the predictor and outcome variables by talking a history of oral contraceptive use and sending a cervical swab to the lab for chlamydia culture. The design of this study is: a. Cohort study c. Case control b. Cross-sectional d. Experimental study 150. a. b. c. d.

What is the first step in the conduct of a clinical trial? Clearly and precisely state the reasons for conducting the trial Clearly define the criteria for selecting the participants Clearly precisely state the objective/s of the trial Make a clear staements of facts governing conduct of the trial of the would be participants

151. The observational study in which subjectys are sampled based on the presence or absence of a risk factor interest, these subjects are followed over the time for the development of a disease outcome: a. Case control study c. Cohort study b. Ecologic study d. Cross-sectional study 152. An investigator wanted to determine if there is a significant difference in the mean systolic blood pressure 10 women before and after oral contraceptive use for 3 months. The appropriate statistical test for significance for this study is: a. Analysis of veriance c. Student t-test b. Chi-square analysis d. Palred t-test 153. a. b. c. d.

In chi-square, one would like to demonstrate that There is a definite cause-effect association There is a significant association among categories There is a significant correlation The number in the 2 x 2 table are diffecrent from one another

a. b.

The objective of data processing is to facilitate Statistical analysis c. Completion of data Termination of research d. Summarizing of data

a. b.

Descriptive statistics include the following EXCEPT: specific rate c. graphical representation measures of dispersion d. tabular presentation

a. b.

The measure that tends to be misleading in the presence of erratic value is Mode c. Median Mean d. All of the above

a. b.

When distribution is symmetrical, the manner of tendency what should be chosen is: Mode c. Mean All of these d. Median

154.

155.

156.

157.

12

158. Randomization is a procedure used for assignments or allocation of subjects to treatment and control groups is experimental studies. Randomization ensures a. That assignments occurs by chance b. That treatment and control groups are alike in all respects expect treatment c. Bias in observation is eliminated d. The placebo effects are eliminated 159. a. b. 160.

When samples are chosen because they are handy and easy this is: Simple random sampling c. Haphazard sampling Systematic sampling d. Cluster sampling

d.

One of these is NOT included in the comprehensive maternal and child health program policies: Pregnant women should be given iron tablets during the second and third trimester and throughout the lactation period Abnormal pregnancies may be delivered at home by a professionally-trained health worker Growth and development of infants and children should be monitored by growth charts and developmental screening devices The newborn receive immediate care at birth and breast feeding instuted within 30 minutes after delivery

a. b. c. d.

Population density is determined by use of two data. One is total population and the other is: Age and sex distribution of the community Area of the community Income of the community Resources in the community

a. b. c. d.

Population would become younger if: Both its crude birth rates and crude death rates remain high Its crude birth rates remain high but its crude death rates decline Its crude birth rates decline but its crude death rates remain high Both its crude rates and death rates decline

a. b.

The population pyramid shows the distribution of a population according to: Age and sex c. Fertility Inflow-outflow equation d. Percentage of births and deaths

a. b.

The arrangement of the population in space in a given time Composition c. Spatal distribution Social mobility d. Size

a. b.

In childhood poisoning, emesis should NOT be induced if the poison is: Barbiturates c. Drain cleaner Acetaminophen d. INH

a. b. c. d.

The following does NOT conform to the 1987 Family Code of the Philippines The contracting parties to a marriage maybe of the same gender Marriage solemnized by a person not legally authorized to perform marriage are void A marriage contracted by any person during the subsistence of a previous marriage shall be null and void Mayor can no longer solemnize marriage

a. b. c.

161.

162.

163.

164.

POISONING 165.

166.

RATES AND RATIO 167. The current priority health problem of the Philippines is: a. Malnutrition c. Rapid population growth b. Communicable disease d. All of the above 168. a. b.

The office that is changed with the function of collecting and reporting of vital health statistics is the: Bureau of Census and Statistics c. Health Information Service DOH Bureau of Medical Services d. Department of Disease Control

a. b.

This is the measure of mortality which will tell us what the 10 leading causes of death rate are: Case fatality rate c. Infant mortality rate Cause specific death rate d. Proportionate death rate

a. b. c. d.

The purpose of immediate notification of disease is to: Study the natural cause of the disease Update the statistical data Prevent the spread of the disease Study the trend of the disease

169.

170.

OCCUPATIONAL HEALTH

13

171. a. b.

One of the following cancers is NOT associated with occupational exposures: Breast CA c. Lung CA Bladder CA d. Liver CA

a. b.

The conservation of health in relation to work, the working environment and assurance of work efficiency is: Industrial hygiene c. Occupational medicine Occupational health d. Industrial medicine

a. b.

Unwanted sound has this physiologic effect on an employee: Deafness c. Inefficiency Irritation d. Autism

a. b.

The primary objective of the practice of medicine according to the Code of Medical Ethics in the Philippines is: Be good citizen c. Be solicitous to patients Be a fiend of man d. Service to mankind

172.

173.

ETHICS 174.

FAMILY MEDICINE 175. The following are important roles of a family physician: a. Personal physician to each member of the family b. Manages the collective health problems of the family c. Recognizes the effects of illness upon the other family members d. All of the above 176. a. b. c. d.

The family life cycle: Depicts the expansion and correction of the family Focuses on major events of developments within a single family Involves a sequence of stressful changes A and B only

a. b. c. d. e.

Measures that the family physician can take to reduce the impact illness on the family are as follows: Do not include the family in deciding on the care of the sick Inform the family of measures important to the success of treatment Watch for change in attitude or negative reaction among family A and B only B and C only

a. b. c. d. e.

The Philippine age structure is considered young. It means A big percentage of the population is made up of the youth More dependents for workers People are retiring young A and B only B and C only

177.

178.

PRIMARY HEALTH CARE 179. The major cause of disabilities in developing countries is: a. Inadequate nutrition c. Accidents b. Infectious disease d. A and B 180.

The first contact of the community to the health chain, as defined by the PHC system is the: a. hospital personnel c. intermediate level health b. village health workers d. barangay captain workers

181. a. b. c. d.

The following are true for volunteer community health workers, EXCEPT: They establish linkage between government and non-government organization They are residents of the community they are community-based they provide only curative care

182. When a community shares in the responsibility and participates in defining the health and health-related problems in the community, this is known as: a. intrasectoral linkage c. appropriate technology b. community participation d. intersectoral linkage 183.

Which of the following is NOT included in the essential health service of Primary Health Care? a. immunization against the major infectious diseases b. provision of essential drugs c. provision of safe water and basic sanitation d. appropriate treatment of all existing diseases

14

184. a. b. c. d.

Which of the following is in line with the principles of Primary Health Care? doctors make all the decisions in a community health program free clinics are held in the population a barangay health council is established to plan, implement and evaluate community program hilots are not allowed to attend deliveries

185. Primary Health Care (PHC) refers to a. the first contact of a patient with a professional health care provider b. a sub-system of the health care delivery system c. a type of health care program designed for communities D. an approach to making health care available and accessible to the population 186. A health care system that has the greatest impact on the health of a society, while making the best use of its resources, fulfills which of the following values of social accountability? A. Relevance C. Cost-effectiveness B. Quality D. Equity 187. Parents who are not qualified as legal dependents, indigents or retirees can avail of PhilHeatlth benefits through the: A. Medicare Para Sa Masa C. Sponsored Projects B. Individually-paying program D. Non-paying members 188. A. B. C. D. Answer

An important feature of a Community-Based Health Program is: preset program objectives managed solely by the people‟s organization people participation in all stages of development availability of modern health equipment for community

#

questions

B

1.

An act instituting a national health insurance program for all Filipinos and establishing the Philippines Health Insurance Corporation. A. Republic Act 7719 B. Republic Act 7875 C. Republic Act 7160 D. Republic Act 6675 MPL: 1

C

2

This is also known as Clean Air Act. A. Republic Act 7277 B. Republic Act 7883 C. Republic Act 8749 D. Republic Act 7600 MPL: 1

A

3

House Bill 618 is also known as: A. National Tobacco Control Act B. Rooming-In and Breastfeeding Act C. Generics Act D. National Blood Services Act MPL: 1

D

4

The National Health Insurance Program (NHIP) shall give the highest priority to achieving coverage of the entire population with at least a basic minimum package of health insurance benefits, the guiding principle is: A. Social solidarity B. Equity C. Compulsory coverage D. Universality MPL: 1

C

5

Mandates iodization of all food grade salt and making this available to all communities nationwide. A. Republic Act 8423 B. Republic Act 8976 C. Republic Act 8172 D. Republic Act 7876 MPL: 1

B

6

page 386

389

403

409

385

Children were hospitalized after their mother fed them with poisonous mushroom. The mother was unable to differentiate poisonous from nonpoisonous species.

15

How would you assess the Meffect of her action? A. Directly voluntary B. Indirectly voluntary C. Positively voluntary D. Negatively voluntary MPL: 0.5 B

7

A patient who has emphysema finds it difficult to quit smoking. How would you assess the morality of the patient‟s action? A. Imperfectly voluntary because of impairment of knowledge B. Imperfectly voluntary because of habit C. Perfectly voluntary act D. An act of man not performed without free will MPL: 0.5

A

8

Which among the following is a human act? A. Cheating in class B. Killing by a known schizophrenic C. An accident caused by severely intoxicated person D. Killing of a wife by the husband who caught her in the act of infidelity MPL: 0.25

A

9

A superior officer not stopping an evil act of a lower rank officer despite knowledge of the same is _________. A. Equally guilty as that of the lower rank officer B. Not guilty of the act C. Guilty but of a lesser degree D. Not morally accountable MPL: 0.5

B

10

Withdrawing food and water in a progressively deteriorating Alzheimer disease patient. Which principle is being violated? A. Beneficence B. Non-maleficence C. Autonomy D. Justice MPL: 0.5

B

11

Which among the following situations is ethical? A. An HIV positive patient continues to engage in unprotected sex. B. Commercial sex workers are mandated to undergo regular screening for sexually transmitted diseases including AIDS. C. Donating blood for a fee D. The owner of an establishment is doing a research on smoking. He asked all his employees to be part of the study. MPL: 0.25

A

12

A 75-y/o diabetic patient with gangrenous foot refused to be amputated knowing all the risks and benefits. She said she is too old to live without a leg. The doctor agreed. What is your ethical evaluation of the doctor‟s decision? A. Right. Patient‟s autonomy is respected. B. Wrong. There‟s harm to the patient. C. Right. Patient accepted an additional burden. D. Wrong. Patient‟s ability to decide is questionable. MPL: 0.5

C

13

A patient told his doctor that he do whatever is best for him. This is not a violation of the principle of autonomy because of the following reason: A. Accepts an additional burden B. Gives up what is due C. Delegate authority D. Looses right to what is due MPL: 0.25

B

14

Principle that emphasizes the freedom to have children A. Autonomy B. Inviolability of life C. Beneficence D. Stewardship MPL: 1

16

D

15

The principle of beneficence means A. To do good B. To do good at all times C. To do good without any harm D. To do good with minimal harm MPL: 0.5

D

16

The A. B. C.

D

17

A 70-y/o quadriplegic patient (paralyzed in all 4 limbs) following repeated strokes requests that no resuscitation be performed if he goes into cardiac arrest. The following would justify a Do Not Resuscitate order EXCEPT A. Resuscitation will cause serious physical and emotional burden B. Resuscitation is against patient‟s autonomous wish C. Resuscitation is extraordinary means D. Resuscitation is proportionate means MPL: 0.25

A

18

The principle of totality is the governing principle in the following situation: A. Plastic surgery and mutilation with due cause B. Voluntary organ donation C. Confinement of mentally against their will D. Conflict of interest MPL: 0.5

B

19

Hospitals cannot be held liable for patients who go home against medical advice because patient A. Gave up what is due B. Lost right to what is due C. Accepted an additional burden D. Delegated authority MPL: 0.5

B

20

following are true EXCEPT Truth telling of harm overrides confidentiality In emergency situation informed consent can be waived Unplugging of life-sustaining machines is justifiable if and when they are no longer useful to the dying person D. Clinical research need not involve animal experimentation MPL: 0.25

A patient in the ward has renal failure. As a junior intern assigned to monitor the patient, you noticed that his „medication is being given in toxic dose. Which virtue is needed in this situation? A. Integrity B. Courage C. Respect D. Honesty MPL: 1

C

21

Which among the following situations is a nonviolation of the principle of justice? A. Kidneys for transplant are scarce. You decided to buy from indigent patients. B. Allocating more of the government funds to kidney transplant program than to the free immunization program for children under six C. Following a typhoon, health care is given to the victim who will die without an aid D. Compulsory tubal ligation in government hospital MPL: 0.5

B

22

Which among the following patients vying for one available respirator would gain the most from using the respirator? A.

Trauma victim with severe brain damage. Only few brainstem functions remain. He is unlikely to recover. B. Patient with infection of the nervous system that rendered him paralyzed from the trunk down. He is, however, progressively improving and is expected to recover. C. Patient with complete and irreversible spinal injury that has left her paralyzed from the neck down. She is able to talk but totally dependent on the respirator. D. Cancer patient undergoing palliative treatment MPL: 0.25

17

C

23

A poor patient needing care agrees to enroll in a research study so as to be admitted in the hospital for free treatment of his medical problem. Was an informed consent obtained in this case? A. Yes. Information is understood by subject. No evidence of constraint on subject‟s voluntariness. B. No. Lack of competence C. No. Questionable voluntariness due to subject‟s personal circumstance. D. No. Probable incomplete disclosure and deception. MPL: 0.5

B

24

An employee was discovered to be sick during the annual physical examination sponsored by the company. His health would adversely be affected if he continues with his job. He pleads not to tell his employer. What should the physician do? A. Maintain confidentiality at all times B. Break confidentiality for the best interest of the patient C. Break confidentiality for the best interest of innocent third party D. Wait and observe MPL: 0.5

A

25

Which among the following is NOT a condition to stop treatment in a dying patient? A. Life is preserved by ordinary means B. Patient and/or family consents C. Irrefutable evidence that biological death is imminent D. Treatment will not prolong life for any significant time MPL: 0.25

D

26

The type of domestic violence, which is considered to be the most common of elder mistreatment, is called A. Physical abuse B. Material exploitation C. Caregiver neglect D. Emotional abuse MPL: 1

D

27

An elderly patient was noted to have hematoma in the legs. Upon probing, patient claimed he fell from the bed. Caregiver was hesitant to bring him to the hospital for medical management. What will you do? A. Report to authorities B. Perform home safety assessment C. Expand social services D. Assess family functioning MPL: 0.5

B

28

Given a patient who is a victim of partner abuse, the following may be done EXCEPT A. Draw a safety plan B. Do couple counseling C. Refer to mental health professional D. Refer to women‟s desk MPL: 0.5

C

29

In cases of sexual assault, the following is recommended: A. Collect clothing and place in plastic bag B. Do a urine pregnancy test C. Give empiric antibiotic treatment for STD D. Insert IUD within 24 hours to prevent pregnancy MPL: 0.25

D

30

Tertiary prevention for those who are victims of violence includes A. Screen for possible violence B. Provide appropriate medical care C. Inform women of their rights D. Refer to local support group and shelter MPL: 1

C

31

Family, as a group of people, is defined in various ways. Which among the following description runs common among the various definitions? A. Related by blood or marriage B. Related by strong affection C. Comprises a permanent household D. Changes through time

18

MPL: 0.25 C

32

Tom and Tina have been living with their aunt and grandmother since their parents went abroad to work as nurses. What is the type of family structure? A. Nuclear B. Extended C. Single-parent D. Communal MPL: 1

B

33

As parents, Jim and Lora make sure that they spend quality time with their children. They play and do things together with them. What function of the family do they perform? A. Biologic B. Psychologic C. Educational D. Socio-cultural MPL: 0.75

C

34

John and Marsha is a family with four children aged 16, 10, 6 and 4. Both husband and wife are in their mid-forties. What second order changes can be anticipated in their family life cycle stage? A. B. C. D. MP:

B

35

Taking on parenting role Sharing responsibilities with extended family Focusing on marital and career issues Keeping communication system open 0.25

Linda was diagnosed with myoma last 2004. Recently, she developed vaginal bleeding. She was told she needs to undergo hysterectomy. She was subsequently observed to be crying and socially withdrawn. Identify the stage in the illness trajectory. A. Reaction to diagnosis B. Major therapeutic efforts C. Early adjustment to outcome D. Adjustment to permanency of outcome MPL: 1

D

36

Which among the following statements is NOT true? A. For every disease, there is a corresponding impact of illness. B. For illness with acute onset, the family is more prone to a crisis situation. C. For stages 2 & 5 of the illness trajectory, the family will go through the same process of denial, anger, and depression. D. For chronic illness, the family is less prone to stress due to coping through time. MPL: 0.25

A

37

Mrs. D began having difficulty remembering things, missing at time of work, and appearing in public in an untidy state at age of 64 years. Mrs. D was brought to a doctor for consult and was diagnosed to have Alzheimer‟s disease. Because Mrs. D‟s condition seemed to progress rapidly, the attending physician did a home visit. When he arrived, she found Mrs. D unkempt, bedridden and has lost weight. The daughter who lives with her complained of exhaustion and of getting very little help from her siblings. In assessing the condition of the index patient and the family, the doctor made use of several tools. Which among the following tools and their corresponding indication for use is NOT correct? A. APGAR – assess familial resources B. Genogram – identify alternate caregivers C. SCREEM – assess capacity of family to participate in health care D. Family map – describe family dynamics MPL: 1

D

38

A 42-y/o woman, married with 2 daughters from Masbate revealed a 2-yr. history of

19

breast mass. Physical examination showed a cachectic patient with unilateral multiple beast mass with foul smelling discharge. The patient sought consult with faith healers in their hometown. Her family and relatives firmly believe that it was due to “barang” (witchcraft). She was brought to Manila to consult with another faith healer. She refused to see a doctor, as it would render ineffective the treatment being done to her by the faith healer. Deep in her heart she has given up hope for cure. For a doctor to understand the health beliefs and practices of the family and the index patient, what tool in family assessment should be used? A. Clinical Biographies B. Family Circle C. Ecomap D. SCREEM MPL: 1 C

39

In evaluating the potential crisis once the patient dies, which among the following would help evaluate the degree of disruption in the function of the family? A. Family B. Family C. Family D. Family MPL: 0.25

stressor developmental stage role coping history

A

40

What would consists tertiary level of prevention for the family? A. Hospice care B. Counseling C. Health education D. Breast examination MPL: 1

A

41

It is essential health care based on practical, scientifically sound and socially acceptable methods of technology made universally accessible to individuals and families in the community and through their full participation and a cost that the community and country can afford to maintain at every stage of their development in the spirit of self reliance and self determination. A. Primary Health Care B. Primary Care C. Secondary Care D. Tertiary Care MPL: 1

C

42

An evaluation or appraisal of present conditions and existing resources is A. Planning B. Assessment C. Situational analysis D. Implementation MPL: 0.25

181

D

43

Demographic factors in planning includes description of A. Existing health services B. Working health systems C. Socio-economic-environmental conditions D. Age-gender composition and distribution MPL: 1

181

D

44

Health status evaluation includes the following, except A. Infant mortality rate B. Hospital services utilization C. All of the above D. None of the above MPL: 1

181

B

45

A gap between what is and what should be A. Idea B. Problem

255

20

C. Priority D. Condition MPL: 0.25 In the Primary Health Care approach, focus group discussion and community assembly are strategies for: A. Appropriate technology B. Networking and linkages C. Establishing support groups D. Community participation MPL: 0.25

D

46

89

B

47

Working together with the people, learning their language and eventually establishing a herbal-medicinal garden for people consumption is an example of A. Establishing support groups B. Appropriate technology C. Networking and linkages D. Community participation MPL: 0.25

C

48

An A. B. C.

B

49

Among the list, the program that would benefit the greatest number of people: 305 A. Provision of food and drugs B. Accessibility to safe water C. Health education D. Proper waste collection and disposal MPL: 0.25

D

50

In the course of implementation, when the quality of service is not at par, management should: A. Review established standards B. Reprimand staff concerned C. Measure difference in output D. Modify activities/objectives MPL: 0.33

A

51

During an evaluation procedure, complaints against nurses have been recorded 342 and analyzed. In making decisions, management should be reminded of: A. Error of practical significance B. Statistical data as evidence based materials C. Opinions of co-managers and staff D. Generalization based on records at hand MPL: 0.25

D

52

Epidemiology can be defined as the study of:

189

example of establishing support groups for program continuity 263 Committing civic, religious, non-government organizations to actively participate Putting up a Botika sa Barangay with the help of the Municipal council Creation of an organization of asthmatic patients in the early identification and proper referral of asthmatics D. Consultative meetings with community leaders for the creation of latrines in certain areas MPL: 0.25

344

A. The etiology of disease in humans B. The frequency of causes of death in humans C. The determinants of frequency pf disease in humans D. The distribution and determinants of frequency of disease in human populations MPL: 0.25 B

53

The time interval between entry of an infectious agent into a host and the onset symptoms is called A. The communicable period B. The incubation period C. The preinfectious period D. The noncontagious period MPL: 0.25

A

54

Primary prevention may be best undertaken during the period of: A. Pre-pathogenesis B. Pathogenesis C. Resolution or sequelae

21

D. Any of the above MPL: 0.33 D

55

In the study of the cause of a disease, the essential difference between an experimental study and an observational study is that in the experimental investigation A. The study is prospective B. The study and control groups are of equal size C. The study and control groups are selected on the basis of history of exposure to the suspected risk factor D. The investigators apply an intervention to influence the outcome of the study, for effective methods of treatment, prevention, or clinical management MPL: 0.25

B

56

The occurrence of a group of illnesses of similar nature at a rate above the expected number is called: A. Hyperendemic B. Epidemic C. Endemic D. Pandemic MPL: 0.5

D

57

To determine whether maternal deficiency of folate is a cause of congenital defects of the neural tube, the mothers of 100 newborns with congenital neural tube defects and 200 newborns without congenital neural tube defects were questioned about intake of multivitamins and folate during pregnancy. What type of study is this? A. Clinical trial B. Cross-sectional C. Cohort D. Case-control MPL: 0.25

A

58

A study revealed an odds ratio of 3.35 (95 % Confidence Interval= 1.35-8.42) associated with maternal deficiency of folate. If the study described is accurate, which of the following statements is true? A. Results suggest that a baby whose mother had folate deficiency is about 3.35 times as likely to be born with congenital defects of the neural tube as a baby whose mother did not have folate deficiency and the association is significant (p 2SD Which of the following is used to show probable relationship between two quantitative variables? A. Component bar diagram C. Bar graph B. Scatterpoint diagram D. Pictogram Which of the following is an alternative to the pie chart? A. Bar graph B. Component bar diagram

C. Frequency Polygon D. Histogram

Which of the following is used to portray absolute or relative frequencies, population rates, or other numerical measurement across the categories of a qualitative or a discrete quantitative variable?

30

A. B. Recall 42.

Recall 43.

Recall 44.

Recall 45.

Recall 46.

Recall 47.

Recall 48.

Recall 49.

Bar graph Line Diagram

C. Frequency polygon D. Histogram

Which type of data presentation is appropriate if the intention is to show detailed information? A. Textular presentation C. Graphical presentation B. Tabular presentation D. All of the above Which of the following is/are requirement/s for Questionnaire A. it should have sensitivity, specificity, and positive predictive value B. respondents are not expected to know all the answer to the questions C. should be ambiguous D. fairly offensive Secondary data are collected by which of the following methods of data collection? A. review of literature/documents C. inquiry B. survey D. observation Chronologically, which of the following steps in Questionnaire Preparation should be last? A. List the variables to be measured C. Prepare the draft of the question B. Pretest the questionnaire D. Make necessary modification If death occurred within the jurisdiction of the Philippines but the exact place of it‟s occurrence cannot be determined, the death shall be registered in the office of the local civil registrar of A. place of burial upon the presentation of a certificate B. victim‟s last known address upon the presentation of a certificate C. birthplace of the victim upon the presentation of a certificate D. none of the above Which of the following should be accomplished if a less than 7 months fetus dies 24 hours after birth? A. birth certificate only B. death certificate only C. both birth and death certificates D. both birth and death certificates marked for Statistical Purposes Only

The process of obtaining a sample where the probability of inclusion of a unit is not known specified is observed in which of the following sampling methods? A. Simple Random sampling C. Stratified Sampling B. Systematic sampling D. Quota sampling Importance of appropriate determination of sample size include/s: A. the expense ,time and work is increased B. the feasibility of the study in terms of availability of the subject is assured C. the duration of study and estimate of the budget can be extended beyond the estimates. D. the precision of the study is diminished

Application 50. An investigator wants to undertake a study involving the incidence of drug abuse among Filipinos, which of the following methods is the most appropriate sampling method to be used? A. Simple Random Sampling C. Systematic Sampling B. Stratified Sampling D. Multistage Sampling Recall 51.

Recall 52.

Living disease agents that readily die outside the human host require for their successful transmission requires: A. an intermediate host C. a common vehicle B. a vector D. the most direct contact Herd immunity depends on the following factors A. degree to which an affected individual is capable of transmitting the infection B. the length of time during which an individual is infectious

31

C. D. Recall 53.

Recall 54.

Recall 55.

Recall 56.

Recall 57.

Recall 58.

Recall 59.

Recall 60.

Recall 61.

Recall 62.

Recall 63.

size and social behaviour of the community all of the above

Characteristics of passive immunity A. it is temporary B. it usually lasts 6 months in the newborn C. it protects an individual for life D. A and B It is the first stage in the natural history of disease wherein disease has not yet developed although the groundwork has been laid through the presence of factors that favor its occurrence A. Prepathogenesis C. Early clinical stage B. Early Pathogenesis D. Late clinical stage The level of prevention that is aimed at intervening before pathological changes have begun during the natural history stage of susceptibility A. Primary prevention C. Tertiary prevention B. Secondary prevention D. none of the above A pattern of disease occurrence in a community where the greater proportion of the cases are subclinical illustrates the A. Interference phenomenon C. Berksonian bias B. Iceberg phenomenon D. Pearl‟s fallacy

Certain diseases that do not confer immunity affect all age groups but the incidence rates are generally among the very young and the very old because A. they generally are more exposed to such diseases B. they generally have lower general resistance C. they have had no experience with such diseases D. immunization protects them from acquiring the disease Diseases which confers long lasting immunity are highest at A. younger age groups C. older age groups B. middle age groups D. very young and very old age groups The most important personal variable related to disease occurrence is A. age C. marital status B. sex D. socio-economic status Death rates are higher for males than females throughout life and this may be due to the following A. sex-linked inheritance C. differences in hormonal balance B. environment D. all of the above The A. B. C. D.

M:F ratio for Acute Myocardial Infarction mortality declines by age 40 because females lost the protective hormones females live longer more females than males by this age and after females seek medical care more than the males

The most direct measure of socio-economic status A. occupation B. family size

C. family income D. area of residence

Observed differences in disease frequency can be due to errors of measurement in which of the following? A. differences in access to medical care C. physiologic differences B. differences in precision of diagnosis D. A and B only

Recall

32

64.

Recall 65.

In which particular situation will you suspect the disease to be a “place” disease? A. The disease has a high occurrence in the native inhabitants but not among the immigrants. B. The disease has a high occurrence in both the native inhabitants and the migrants to the place. C. The disease will continue to have a high occurrence among the offspring of the native inhabitants who have migrated out of the area. D. The disease has a low occurrence among the native inhabitants and a high occurrence among the migrants. In the Philippines, generally, variation in mortality rates between the cities and the provinces are due to differences in: A. genetic susceptibility of the populations B. exposure to the risk factors C. availability and accessibility of medical care D. accuracy and completeness of diagnosis and reporting

Analysis 66. Which of the following can best explain the higher occurrence of breast cancer among the second generation Chinese, Japanese and Mexican migrants than the first generation migrants? A. The second generation migrants lost the protection provided by their genetic make-up. B. The first generation migrants partly retained their traditional habits and practices. C. The second generation migrants were exposed to the risk factors for a longer period of time. D. Breast cancer has a long latency period and therefore, would be expressed more in the second generation migrants. Recall 67.

Recall 68.

Recall 69.

Recall 70.

Recall 71.

Recall 72.

High herd resistance to an infectious disease limits the occurrence of the disease in a community because: A. It prevents exposure of the population to the disease. B. It blocks the transmission of the disease. C. It shortens the transmission time. D. It reduces the chances of death among the cases. Changes in disease frequency usually encompassing several decades are referred to as: A. Periodic trends C. cyclic intrinsic variations B. secular trends D. cyclic extrinsic fluctuations If the line graph of mortality of a particular disease showed an abrupt dip or decrease, this would most likely be due to: A. an increase in herd resistance of the population B. an improvement in diagnostic procedures C. an improvement in treatment modalities D. altered procedures in assigning causes of death Higher patterns of non-infectious diseases during certain seasons are probably due to A. accumulation of susceptibles B. change in the climate facilitating multiplication of disease agent C. change in climate modifying human activities E. change in disease reporting A Case-report is commonly done for A. statistical and registration purposes B. legal purposes C. documenting an interesting medical oddity D. generating hypothesis Which type of study determines/measures both the exposure or factor and the effect or outcome at the same time? A. Case-report study C. Cross-sectional study B. Case-series study D. Ecological study

Analysis 73. Which of the following study is appropriate if an investigator wants to compare the 2 and 5 years survival rates of leukemia if the subjects are randomly given a certain drug X? A. Case Series studies C. Case Control studies B. Cohort studies D. Experimental studies

33

Analysis 74. Which of the following is most appropriate if the objective of the study is to determine if hypotension increases the chance of fatal outcome in MI patients A. Case Series studies C. Case Control studies B. Cohort studies D. Experimental studies Recall 75.

Recall 76.

Recall 77.

Recall 78.

Recall 79.

Recall 80.

Recall 81.

Recall 82.

Recall 83.

Recall 84.

The A. B. C. D.

initial step in the analysis of data in case-control studies is determination of the scale of measurement assessment of the factor computation of the indicators assessment of causality

The A. B. C. D.

appropriate indicators in Case-control study is the frequency/prevalence of the independent variable in both the cases and controls the frequency/prevalence of the dependent variable in the exposed and unexposed groups the measure of risk of occurrence existence of statistical association

Statistical association found between a factor and disease could be causal if the association is strong; the strength of association in Case-control studies is measured by the A. Relative risk C. Odd‟s ratio B. Attributable risk D. Risk ratio Which of the following statements is true about cohort studies? A. They are also called prevalence studies. B. Subjects are chosen on the basis of whether they have the outcome or not. C. They begin with identification of a risk group. D. Actual risk of the outcome cannot be determined using cohort studies. Which is an advantage of a concurrent over a non-concurrent cohort study? A. smaller sample size C. less biases B. shorter follow-up period D. direct measurement of outcome A requirement for validity of cohort studies is that: A. subjects should be representative of target population. B. exposure variable should be randomly distributed between the study groups. C. study groups should be similar in all respects except for the exposure. D. investigator should be blinded to eliminate bias. A variable other than the one under investigation which may be associated with both the factor and the outcome and thus may produce a spurious association is called a/an: A. independent variable C. confounding variable B. dependent variable D. misclassification variable Which of the following is not a requirement of a descriptive study? A. Sample representative of target population B. Controls C. Similarity of comparison groups D. All of the above

Which of the following is TRUE with regards to the subjects of an Experimental study? A. Sample representative of target population B. well and diseased persons can included in the sample in a trial of preventive measure C. diseased persons can be the study population in a curative trial D. all of the above Which of the following is a strength of Experimental study?

34

A. B. C. D. Recall 85.

Recall 86.

Recall 87.

Recall 88.

Recall 89.

Recall 90.

Recall 92.

Recall 93.

Recall 94.

Recall 95.

Recall 96.

appropriate for diseases with low or rare occurrence can study multiple effect of a single exposure allows artificial manipulation of the study factors all of the above

Which of the following statement is TRUE? A. Blinding aim to eliminate the effects of intervention B. Placebo administration aim to eliminate the effects of unintended factors C. Both blinding and Placebo administration aim to eliminate the effects of intervention D. Both blinding and Placebo administration aim to eliminate the effects of intended factors The strongest of all research study is: A. Uncontrolled Trial B. Parallel Controlled Trial

C. D.

Cross-Over Trial Quasi-experiment

In an experimental study, if groups show dissimilarity, adjustments should be done in the: A. Measurement of baseline variable C. measurement of outcome variable B. Application of treatment D. analysis of data Which of the following is most vulnerable to Hawthorne effect? A. Parallel controlled trial C. Uncontrolled trial B. Cross-over trial D. None of the above Which of the following trial should be use to determine the effectiveness of a health program? A. Community trial C. Therapeutic trial B. Clinical trial D. Prophylactic trial Dengue is currently occurring in epidemic proportion in several areas in the country because A. the existing strain of the disease agent has become more virulent B. a new strain of the disease agent has been introduced C. of increased opportunities for transmission D. of inadequacy of health services To establish the existence of an outbreak/epidemic, which of the following should be done? A. construct the epidemic curve B. determine the onset of the epidemic C. compare the present incidence with the normal expectancy D. compute and compare the attack rate Reducing or exhausting the susceptibles in the area to prevent new cases from developing would be accomplished if A. they are quarantined B. they are attacked by the disease C. the cases are isolated D. the cases are adequately treated If quarantine is necessary to contain an outbreak/epidemic, who of the following should be quarantined? A. Cases C. contacts B. Carriers D. suspects Which type of study is generally utilized in investigating outbreak/epidemics? A. case series C. cohort B. case-control D. interventional The initial step in planning for data collection is: A. Determine the research objectives C. Specify source of data B. Identify the data to be collected D. Choose the appropriate method

Recall

35

97.

Recall 98.

Recall 99.

Recall 100.

Planning for data collection is based on the: A. research objectives C. number of subjects B. data to be collected D. time for data collection The main consideration in the choice of the method of data collection is the: A. ability of the method to yield measurements or values that is closest to the true value B. availability of resources for data collection C. acceptability of the method to the subjects D. ability of the method to achieve good coverage of the study population The probability that a test will declare positive those who have the disease is: A. sensitivity C. positive predictive value B. specificity D. positive likelihood ratio Characteristic of selection of appropriate and adequate subjects includes: A. A vital stage in any research activity. B. Begins with determination of suitable reference or target population and ends with response of the selected persons to the invitation to join the study. C. It influences the validity of the study. D. All of the above

B___1. Adopted children are considered members of: A. Blended family B. Nuclear family C. Extended family D. Consanguineal family B___2. Community interaction and independence are encouraged in this family type: A. Random type family B. Open type family C. Close type family D. Corporate family B___3. This family assessment tool enables the physician to determine family dynamics and family health problems: A. Family Genogram B. APGAR C. SCREEM D. Family Circle A___4. Present day Filipino families have accepted this family phenomenon: A. Husband no longer monopolizes as the bread winner B. Institutionalization of the aged member C. Parenting is restrictive D. Authoritative parents A___5. Family Life Cycle provides the physicians the most important information regarding: A. sequence of stressful events with family‟s readjustments to maintain health and viability B. weak points are identified for the physician to strengthen C. bad practices for the physician to correct D. the dominant person in the family is identified for the physician to solely rely on him/her C___6. Considered most challenging stage of the family life cycle : A. Unattached young adult t B. Newly married couple C. Family with young children D. Family with adolescent D___7. This stage of family Life Cycle is characterized by the shifting generational roles: A. Newly married couple B. Family with adolescent C. Launching family D. Family in later years D___8. A time for positive growth and creative exploration for the entire family A. Unattached young adult

36

B. C. D.

Newly married couple Family with young children Family with adolescent

C__9 . The most important aspect of health promotion among adolescents A. Screening B. Immunization C. Health education D. Counseling

A__10. Almost 50% of pregnant Filipino women are suffering from:: A. anemia B. scurvy C. vitamin deficiency D. lack of iodine C__11

Median urinary iodine level that indicates mild IDD A. < 20 ug/L B. 20-49 ug/L C. 50-99 ug/L D. > 100 ug/L

C__12. Daily vitamin A. B. C. D. E.

A intake of pregnant women should not exceed 500 iu/day 5,000 iu/day 10,000 iu/day 20,000 iu/day 200,000 iu/day

B__13 Most frequently reported debilitating chronic disorder among the elderly: A. asthma B. hypertension C. diabetes mellitus D. arthritis C__14 Triage is done A. B. C. D.

during this phase of the Emergency Medical Services system: pre-treatment phase preliminary care phase definitive care phase at the receiving facility recuperation phase

A___15. Which of the following is the real primary target of National Immunization Day? A. Poliomyelitis B. Tuberculosis C. Measles D. Diptheria, pertussis, tetanus D___16 The A. B. C. D.

main goal of the country‟s CARI program is: Early detection of pneumonia cases Prompt treatment of Pneumonia cases Efficient clinical assessment and management of Pneumonia cases Reduction of Pneumonia deaths

A__ 17. To lessen mortality from pneumonia, the CARI program endorses: A. early diagnosis of pneumonia cases B. hospital admission of all patients with pneumonia C. giving oral antibiotics to all children with cough and colds D. influenza immunization C___18. With the emergence of cardiovascular as the leading cause of mortality, DOH formulated the National Cardiovascular Disease Prevention and Control Program aimed primarily to: A. identify high risk individuals B. treat all identified cases of cardiovascular disease C. promote healthy lifestyle D. promote the use of indigenous drugs to treat cases

37

D__19. Inaccessibility to health care in the Philippines is mostly due to: A. migration of doctors to other countries B. lack of health centers and barangay health stations C. lack of nurses D. most hospitals, health care facilities and doctors are urban based B___20. Increased prevalence rate is seen in: A. increasing population B. increased survival C. increased incidence D. epidemics B___21. A stage in the Family Life Cycle where parents and grandparents are encountering crisis as well: A. Launching B. Family in later years C. Family with adolescents D. Family with young children A.__22. An example of primary prevention is: A. Measles immunization B. self examination of the breast for tumors C. smoking cessation after a heart attack D. cervical cytology screening B__ 24. Births in the Philippines are registered in the municipality or city : A. where the infant‟s parents live B. where the infant was born C. of the parent‟s choice D. listed as the family‟s permanent residence „ C __25. The cause of death coded for the purpose of tabulating mortality statistics is: A. Immediate cause of death B. intermediate cause of death C. underlying cause of death D. antecedent cause of death B__26 . The registration of death in the Philippines as specified by law must be done not later than: A. 24 hours after death B. 48 hours after death C. 2 weeks after death D. 30 days after death C__27. Deaths in the Philippines are registered in the municipality or city:: A. where the deceased was born B. of the person‟s usual residence C. where the person died D. listed as the person‟s permanent address A__28 Accumulation of this metal in organs and tissues of humans has been linked to brain damage: A. mercury B. aluminum C. lead D. silver B__ 29 This CFC offender is “ozone safe” and does not contain chlorine: A. HCFC (hydrochlorofluorocarbons) B. HFCs (hydrofluorocarbons) C. Hydrocarbons D. Chlorofluorocarbons

B__ .30. Method of food preservation which prevents multiplication of bacteria without destroying them: A. preserving

38

B. C. D.

refrigeration fermentation pickling

C__31 . Most important test to determine if the water is potentially dangerous: A. physical examination B. biological examination C. bacteriological examination D. chemical examination B___32. The coliform count from a well is 35/100 cc. The treatment required is: A. Rejection of sources B. Standard treatment C. Double treatment D. Triple treatment A___33. Milk is best preserved by this method: A. Sterilization B. Boiling C. Freezing D. Adding sodium benzoate C___34. Of the following devices for excreta disposal, the most dangerous in contaminating ground water is: A. Chemical privy B. Pail system C. Bored hole latrine D. Septic tank B___35 The concentration of which of the following greenhouse gases is affected by deforestation A. Water vapor C. Methane D. Nitrous oxide

B. Chlorofluorocarbons

B___36. Formation of sticky precipitates of soap with water indicates that the water is: A. Polluted B. Hard C. Contaminated D. Soft D___37. Increase in which of the following in drinking water is associated with Alzheimer‟s disease A. Antimony B. Zinc C. Silver D. Aluminum B___38. Dissolved gases and odors in water are removed by: A. Filtration C. Sedimentation B. Aeration D. Coagulation D___39. Chlorination of water is directed towards: A. Bad taste C. Turbidity B. Odor D. Bacteria B___ 40. The recommendedl refuse disposal method for hospital is: A. Dumping B. Incineration C. Recycling D. Composting A___41. The cheapest and most frequently used refuse disposal in the community but can result to water and air pollution is: A. Dumping B. Sanitary landfill C. Incineration] D. Reduction and salvage C__42. The first step in an epidemiologic/outbreak investigation; A. Formulate the hypothesis B. Characterize the epidemic

39

C. D.

Determine the existence of an epidemic Implement control methods

B__43. The ultimate goal of an epidemiologic investigation: A. To request for international aid B. Institute curative, preventive and control measures to avoid more cases C. Identify administrative and operational problems of a health agency D. To determine discrepancies or gaps in policy implementation A___44. Primary Health Care Concept is: A. meant for people to attain health by being responsible for it B. the provision of all health care services at affordable prices C. meant to give accessible, affordable and available primary, secondary and tertiary care D. a level of care B__45. Direction of personnel and techniques in operation as a component of public health administration is: A. organization B. management C. staffing D. performance evaluation A__46 . Community Diagnosis is usually regarded as: A. situational analysis B. analysis of needs C. health survey D. health program planning C__47. In the model A. B. C. D.

of a Health Care Delivery System, “input” refers to: health activities health products health resources feedback

A__48. Regulation of importation, manufacture, marketing, and consumer utilization of all drugs refer to: A. quality control B. rational use of drugs C. development of self-reliance in the local pharmaceutical industry D. tailored or targeted procurement of drugs by the government D__49. Control as a component of a health system model is exemplified by: A. immunization programs B. number of fully immunized children C. vaccines and reagents D. immunization protocol C___50. In community health planning, the BEST people to identify and prioritize problems are the: A. Community doctors D. Social worker B. Volunteer health workers E. Nurses/midwives C. Community people themselves C__51. Recognition of each one‟s roles and functions in a multidisciplinary team is A. Comprehensive B. Continuous C. Teamwork D. Coordinated B__52. In a planning cycle, analysis of why programs and projects fail is: A. Situational analysis B. Evaluation C. Impact analysis D. planning A__53. With the implementation of the Local Government Code, municipal and city health services are placed under the supervision of the: A. City and municipal mayors B. Doctors of district hospitals B. DOH C. PGH

40

D__54. Which of A. IMR B. IMR C. IMR D. IMR

the following will make a country a priority for international aid? of 5/1000 livebirths of 15/1000 livebirths of 20/1000 livebirths of 50/1000 livebirths

B__55. The ideal denominator for Maternal Mortality Rate but cannot be used : A. Total livebirths B. Number of pregnant women C. Total births D. Midyear population A__56. Most A. B. C. D.

commonly used index of fertility: Crude Birth Rate General Fertility Rate Age Specific Birth Rate Total Fertility Rate

C__57. Regarding exposure and outcome a relative risk of 15 for a certain disease means: A. Non causal association B. Beneficial association C. Harmful association D. Definite causal association B__58. Swaroop”s index in an indicator of the following: A. B. C. D. B__59. The A. B. C. D.

medical , social and economic condition of the area where it is applied a better chance for longer survival given to a population relative importance of different diseases as cause of death the “killing power” of a disease first line of defense against occupational hazard: personal protective equipment engineering control ergonomics good nutrition

A__60. For asymptomatic and normotensive elderly, blood pressure monitoring should A. B. C. D.

be: Every month Every 6 months Every year Whenever convenient

A__61. For high risk elderly women Pap‟s smear should be done: A. B. C. D.

Annually Every 2 years Every 3 years Every 5 years

C__62. The earliest reliable predictor of pneumonia in the CARI Program is: A. cyanosis B. chest indrawing C. tachypnea D. lethargy B__63. In the CARI Program a child with productive cough must be given: A. Antitussives B. A lot of fluids C. Expoectorants D. Antibiotics C__64. In the CARI Program the diagnosis for a 6-month old child with RR = 60/min, Chest , indrawing, intercostal and subcostal retraction but without cyanosis A. No pneumonia B. Pneumonia C. Severe pneumonia

41

D.

Very severe pneumonia

A__ 65. The first line of antibiotic regimen under the CARI Program: A. oral Co-trimoxazole B. procaine penicillin C. chloramphenicol D. gentamicin B__66 So that an a. b. c. d.

epidemic of chicken pox will not occur the best control measure is: quarantine of cases isolation of cases control of animal reservoir surveillance of cases

D__67. Success in ORT of a child with diarrhea at home is achieved by: A. the doctor explaining to the mother/caretaker how it is done B. having posters on the clinic wall showing how ORT is done C. a nurse or health worker demonstrating ORT D. the mother/caretaker practicing giving ORT guided by a health worker B__68. The main thrust in the control of diarrhea by the DOH: A. environmental sanitation B. oral rehydration therapy with ORESOL C. public education D. immunization A__69. A very early sign of dehydration in a child with diarrhea: A. thirst B. sunken eyes C. increased pulse and breathing rates D. somnolence D__70. Primary reason for inclusion of glucose in oral rehydration solution: A. as a source of calorie B. for palatability C. to increase the nutritive value D. to help in the absorption of sodium B__71. A. proportionate Mortality indicator: A. Age-sex specific death rate B. Swaroop‟s index C. Infant Mortality Rate D. Neonatal Mortality Rate

B__72. To establish ways to improve the overall health status of communities the best experimental approach to use is: A. B. C. D.

Clinical trial Community trial Therapeutic trial Intervention trial

B__73. The most accurate relative risk is the one established in: A. case-control study B. cohort prospective C. cohort retrospective D. cross-sectional study E. all of the above

B__74. To determine the prevalence of a disease this study design is used A. B. C.

Descriptive Cross sectional Case control

D. cohort E. experimental

C__75. A cross-sectional study is not applicable for this purpose: A. determine the magnitude of diseases

42

B. C. D.

establish baseline data study causes of disease study rare diseases

D__76. Controls in a case-control study are those: A. who will constitute the comparison group B. who will not develop the outcome C. who will not be given the experimental drug D. who do not have the illness being studied

C__78. The most important thing to consider in the selection of cases in a case-control study: A. B. C. D.

That That That That

the the the the

cases cases cases cases

will definitely show positive exposure can be interviewed regarding past experiences satisfy the specific diagnostic criteria are typical of the patients to be studied

C__79. Not true of cohort studies: A. Investigators still have to wait for the outcome B. Controls are those without exposure or risk factor C. Intervention is given to the exposure group D. Study moves forward with time C__80. Statement/s that describes an experimental study: A. The investigator can only observe occurrence of disease in people under natural setting B. Contrast in outcome between study groups are observed, not created in a controlled way C. The investigator controls conditions under which the study is done D. The investigator observes disease occurrence in people who are grouped on the basis of experience or exposure.

D__81. The study design where the investigator has control over the conditions of the study: A. B. C. D.

cross-sectional case-control cohort experimental

l

D__ 82.Experimental studies are concerned with A. B. C. D.

Investigating hypothesis regarding causal association Measuring effects on animals of introducing in a controlled way new cures for diseases Observing distribution of diseases in population Testing hypothesis through direct control over assignment of subjects to study groups

D__ 83. Intervention trial is exemplified by: A. B. C. D.

Simple mastectomy for breast cancer BCG vaccination for prevention of TB Isoniazid for TB prophylaxis Antihypertensive drugs to minimize stroke

43

B__ 84. The probability sampling method that assures adequate representation of minority sub-groups A. Systematic sampling B. Stratified sampling C. Cluster sampling D. Simple random sampling : C__85. Management of infants with diarrhea under the CDD Program A. Rest the stomach by fasting for a day B. Shift from breastfeeding to lactose free formula C. Add vegetable oil to food D. Start co-trimoxazole after 6 episodes of diarrhea D__86. The most effective method to control TB in the Philippines is: A. Isolation of TB patients B. Mass chemoprophylaxis with INH C. Mass chest x-ray and treatment of cases D. Mass BCG vaccination B__ 87.The most common complication associated with BCG vaccination is: convulsion suppurative lymphadenitis encephalitis reactivation of TB B__ 88.The management for an infant who developed a 10 mm red, tender swelling at the BCG site is: A. application of topical antibiotic on the swelling B. assurance of the mother that no treatment is needed C. application of pulverized INH on the swelling D. prophylactic INH at 10 mg/kg/day C__89. Occurrence of a group of illness of similar nature clearly in excess of normal expectancy; A. Holoendemic B. Hyperendemic C. Epidemic D. Pandemic C__90 The rate used as an indicator of an outbreak or an epidemic: A. Morbidity rate B. Case fatality rate C. Incidence rate D. Prevalence rate A__91. A graphical presentation of data most useful in locating cases during an epidemic: A. Spot map B. Pictograph C. Scatter diagram D. Histogram C__92. Risk considered due to exposure: A. B. C. D.

relative absolute attributable odd‟s ratio

A__93. The one that establishes cause-effect relationship is: A. relative risk B. absolute risk C. attributable risk D. odd‟s ratio B__94. Randomized control trial means: A. randomization was used and there is a control group B. randomization was used, there is a control group and conditions are controlled by the investigator C. randomization was used, there is a control group and the study is blinded D. randomization was used, there is a control group, the study is double blinded D__95. Cross-over trial means:

44

A. B. C. D.

with separate comparison groups with separate control groups no control group subject and control are the same persons

D__96. To encourage community participation, the local doctor should do the following: A. invite people from civic organizations B. refer to specialist if the need arises C. encourage sponsorship of programs D. listen when people relate their problems B__97. In the Philippines, the age range most infected with HIV is: A. 15 – 24 years old B. 25 – 39 years old C. 35 - 44 years old D. 45 – 55 years old B__98. The most important mission of the Department of Health A. Promote healthy lifestyle B. Ensure accessibility and quality of health care C. Reduce morbidity and mortality D. Improve the general health status of the people C__99. Which diagnostic test is constantly reviewed in the National TB program? A. Bronchoscopy B. Chest x-ray C. Sputum culture D. Tuberculin skin test C__100. If a pregnant woman received 3 doses of Tetanus Toxoid she will have an immunity against tetanus for how many years? A. 3 years B. 5 years C. 10 years D. 15 years C__101. The mother of a 5-month old infant claimed that her baby had received complete immunization for his age. One can assume that the baby has already :received: A. BCG and measles vaccine B. BCG, 3 doses of OPV and 1 measles vaccine C. BCG and 2 doses of DPT vaccine D. BCG, 3 doses of OPV, DPT, Hepatitis B vaccine, 1 measle4s vaccine 1. Primary health care as defined in Alma Ata is referred to as: A. first contact between a physician and the patient B. strategy utilizing participatory approach in the provision of basic health services to the people. C. training program on first aid for the people in the communities D. a program of health activities carried out by health professionals 2.

A characteristics feature of Primary Health Care include one of the following: A. community participation B. infrastructure development and modernization C. emphasis on curative medical care D. training of health professionals

3.

People empowerment in primary health care is brought about through this means: A. volunteerism among the target population B. hiring and employment of community health workers C. community organization process D. endorsement from church and political community leaders

4.

This type of health program allows maximum room for community participation and empowerment: A. hospital based program B. community oriented health program C. community managed health program D. community-based health program

5.

The basic foundation of the community based and managed health program is: A. team approach B. health human rights C. equality and justice

45

D.

community organization

6.

The main case finding strategy advocated in the National Tuberculosis Program is: A. chest radiography B. sputum microscopy C. DOTS D. Mantoux test

7.

Based on the national Malaria control program, the currently recommended principal measure directed against the mosquito vectors in Malaria is: A. biologic vector control B. residual spraying with DDT C. use of chemical impregnated mosquito nets D. sterilization of male mosquitoes

8.

The Integrated Management of Childhood Illnesses (IMCI) requires the delivery of the following service(s) for a child who visit a physician‟s clinic for any health related problems: A. routine immunization B. breastfeeding education for the mother C. proper and adequate nutrition counseling D. all of the above

9.

The most cost effective preventive measure for the control of cardiovascular diseases: A. CVD screening using ECG B. environmental cleanliness C. primordial prevention of CVD risk factor D. genetic counseling

10. This is an essential health service that is available, accessible and affordable by the community: A. social care B. preventive medicine C. medical care D. primary health care 11. A. B. C. D.

The following level of health care does NOT belong to primary level: teacher health guardians emergency hospital rural health unit rural health midwife

12. Which of the following measures has the greatest impact on community health? A. provision of social services B. provision of potable water supply C. immunization D. medical care of the sick 13. The health status of a community may be measured or expressed in terms of: A. availability and utilization of health care services B. availability of public utilities C. its statistical indices of morbidity and mortality D. all of the above 14. Community planning should start with: A. formation of objectives B. identification of roles and their relationships C. educational analysis D. determination of resources 15. The local health department is usually associated with this type of health care planning: A. morbidity planning B. population-based planning C. institution-based planning D. financial planning 16. The prime yardstick of child health level in the community is A. infant and child mortality rate B. number of live births C. number of preschool children D. number of health centers

46

17. Growth A. B. C. D.

monitoring is important in a primary health care program because it tells us how many children are at risk of being malnourished affords an occasion for maternal education tells us how many children are undernourished in the community improves the nutritional status of children in the community

18. Which factor determines the highest use of health care? A. sex B. education C. age D. income 19. The lowest operating unit of the DOH is the: A. emergency hospital B. rural health unit C. barangay health station D. district health office 20. Public health centers do NOT need constant supplies of: A. antidiarrheals B. analgesics C. antipyretics D. antibiotics 21. A primary health care team is composed of: A. Office of the Secretary of Health B. Provincial Health Officer and staff C. Municipal Health Unit and staff D. Interdisciplinary composition of community people working to gather under professional supervision 22. The principal and unique strategy of the PHC is: A. empowerment of the people B. holistic health care delivery C. teamwork among health personnel D. socialization of health care 23. Which of the following is NOT included in the essential health service of Primary Health Care? A. immunization against the major infectious disease B. provision of free drugs C. provision of safe water and basic sanitation D. appropriate treatment of all existing diseases 24. Medicare benefits include: A. psychiatry B. surgery C. optometry D. diagnostic tests 25. Under Medicare and most hospitals, the anesthesiologist's fee should NOT exceed _____% of the allowable surgeons' fee: A. 10% B. 30% C. 40% D. 20% 26. The 1995 National Health Insurance Program covers the: A. government employees only B. indigent patients only C. all Filipino citizens D. unemployed citizens 27. BCG vaccine is given according to the Expanded Program on Immunization of the Philippines, at what age? A. 3 months B. 6 months C. at birth D. 9 months 28. Control TB infectivity is best achieved by: A. isolation B. personal hygiene and cleanliness C. prompt specific drug treatment

47

D. 29. The Department A. B. C. D.

education of Health currently recommends the most effective mode of anti-TB treatment by: two-week monitoring directly observed therapy home visits health center based follow-up

30. During a diarrhea bout, breastfeeding of this infant should be A. stopped and substitute with a lactose-free formula B. temporarily discontinued C. given more often D. limited 31. A program in the A. B. C. D.

control of communicable diarrheal diseases by the Department of Health: hydrotherapy oresol therapy physiotherapy use of intravenous fluids

32. Screening of children has been proven cost-effective for all of the following, EXCEPT: A. hypertension B. iron-deficiency anemia C. vision impairment D. tuberculosis 33. The Civil registry A. B. C. D.

Law includes the registration of Births and marriages births, sickness and marriages births, deaths, marriages births, deaths, sicknesses, marriages

34. A 28 day old infant is classified as: A. neonatal mortality B. fetal neonatal mortality C. post-neonatal mortality D. perinatal mortality 35. The denominator A. B. C. D.

in getting the sex ratio in a population is number of females number of fertile females number of fertile males number of males

36. An investigator reported three cases of carcinoma of the lungs among workers in a cement plant. This type of study is best described as A. Cohort B. Case series C. Case report D. Case control . 37. The major activities undertaken in the maternal and child health program does NOT include A. Immunization of pregnant mothers against measles B. proper pneumonia prevention C. promotion of prenatal care D. monitoring of growth and health status of infants and children 38. One of the major A. B. C. D.

determinants of the degree to which chance affects the findings in any particular study is statistical testing sample size randomization bias

39. Hepatitis B spread is NOT known to occur in A. eating food B. sexual contacts C. household contact D. mother to child transmission in perinatal period 40. What kind of epidemiologic study should be done to determine the cause of an unusual infection?

48

A. B. C. D.

experimental studies analytic study descriptive study none of these

41. One of the following is NOT descriptive of a young population A. population pyramid with a wide base B. median age of 20-24 years C. crude birth rate greater than 30/1000 D. dependency ratio of 1:1 42. Specific health protection can be carried out by A. Immunization B. chemotherapy C. Health education D. A and B only 43. The Expanded Program on Immunization is concerned with this particular age group: A. Infants below 1 yr, school entrance, and pregnant women B. Infants below 6 mos, pre-schoolers, and pregnant women C. Neonates, pre-schoolers, and post-partum women D. Infants of any age, school children, and pregnant women 44. The recommended age for the start of DPT-OPV immunization under the government EPI is A. 8 weeks B. 10 weeks C. 6 weeks D. 12 weeks 45. An example of artificial active immunity is one that is derived from A. giving of gamma globulin B. vaccination C. actual illness D. transplacental passage of antibodies 46. Controlled studies mean A. An experiment is being done B. There is a comparison group C. the characteristics of the participants are all the same D. a group is not to be given the experimental procedure 47. A study was done to prove that co-trimoxazole is more effective than other drugs in treating respiratory tract infections in ambulatory patients in a community. The suitable control group will receive A. Another drug for respiratory infections B. The standard drug for treating respiratory infections C. another experimental drug for respiratory infection D. a placebo 48. A comparison or A. B. C. D.

control groups is NOT necessary in Prospective studies Retrospective studies experimental studies descriptive studies

49. Results are generalized to a comparable population A. Internal validity B. External validity C. bias D. confounders 50. Double blind determination of the exposure and outcome is for the purpose of A. Reducing the effects of losses to follow up B. Reducing the effects of sampling variation C. achieving comparability of cases and controls D. avoiding observer and subject bias 51.

Describes the distribution of cases by the variables of person, time, and place in order to study and explain acute outbreaks of disease and to develop hypothesis about disease transmission: A. Case control studies B. Analytic studies C. descriptive studies

49

D.

Experimental studies

52.

Descriptive studies do A. Draw inferences about cause and effect B. Predict outcome C. not have a sample size estimation D. not require a hypothesis

53.

The possibility A. B. C. D.

54.

A study comparing the prevalence of hepatitis B infection among government and private hospitals falls under A. Descriptive study B. Cohort study C. case-control D. cross-sectional

55.

A well-defined group of people who have had a common experience or exposure who are then followed up for the incidence of new diseases or events is a A. Variable B. Sample C. Case D. Cohort

56.

Odds ratio is the measure of analysis commonly used in A. Case report B. Case-control studies C. case series D. cohort studies

57.

A study is planned to determine if there are more users of oral contraceptives among patients with thrombophlebitis than among well persons. To achieve the objective, which of the following type of studies is most appropriate? A. Clinical B. Descriptive C. concurrent cohort D. case control

58.

In a cohort study of the association of leukemia with x-ray exposure, the relative risk was found to be 3. What does this mean? A. Those with x-ray exposure have threefold probability of developing leukemia B. X-ray exposure protects the person from developing leukemia C. there is no association between leukemia and x-ray exposure D. none of the above

59.

Which of the following study designs most appropriately illustrates this case history: “A total of 200 newly diagnosed patients with endometrial cancer are allocated to treatment with either surgical removal alone or surgical removal plus radiation treatment A. Clinical trial B. Case-control study C. case series report D. cohort study

60.

The observational study in which subjects are sampled based on the presence or absence of a risk factor of interest. These subjects are followed over time for the development of a disease outcome A. Case-control study B. Ecologic study C. cohort study D. cross-sectional study

61.

In qualitative data, the appropriate measure of central tendency is the A. median B. average C. mean D. mode

62.

"In a frequency distribution, the value that divides the observed values into two parts is the

of cause-effect relationship is difficult to assess in Experimental Cross-sectional cohort case-control

50

A. B. C. D.

median and mode median mean mode

63.

If there is an inherent order or progression among categories, they are said to be measure by: A. numerical scale B. nominal scale C. discrete scale D. ordinal scale

64.

To represent the trend over time in crude birth rate and crude death rate in a given province, the best graph to use is a: A. bar B. histogram C. line diagram D. frequency polygon

65.

The rate that measures the probability of a person dying in a year regardless of age, sex, race, etc is: A. case fatality rate B. neonatal mortality rate C. crude death rate D. specific death rate

66.

The following indices use the midyear population as the denominator, EXCEPT: A. crude birth rate B. measles incidence rate C. crude death rate D. proportionate mortality rate

72. The following is an indirect method of nutritional assessment: A. Weight for height B. Percentage of low birth weight babies C. Height for age D. Measles mortality rate 73. The number of live births reported during a given time interval divided by the estimated number of wo men in the 15-44 year old group at mid-interval is: A. crude birth rate B. incidence rate C. prevalence rate D. fertility rate 74. This is the most commonly used index of fertility: A. crude death rate B. infant mortality rate C. maternal mortality rate D. crude birth rate 75.

Turbidity of a given water is due to the: a. radioactive contaminant in water b. total mineral content c. substances in solution d. impurities in suspension

76. Hardness of water is primarily due to: a. calcium and magnesium b. nitrogen content c. iron content d. sulphur content 77. Laws and regulations on Food Sanitation are implemented to protect the public against fraud and deceit. Some of the restraints are the following, EXCEPT: a. misbranding b. sale of damage food c. food adulteration d. adequate food storage 78. Which of the following is the primary objective of food sanitation? a. to ensure the consumption of safe and wholesome food b. to prevent the sale of food which is inferior in value and quality c. to cut down spoilage and wastage of food

51

d.

none of the above

79. Excreta disposal is considered to be sanitary if a. there is minimal fecal handling b. it prevents soil and water contamination c. it is accessible and acceptable contamination d. it prevents both soil water contamination and there is minimum fecal handling 80.One of the following is not a. b. c. d. 81.Sanitary methods a. b. c. d.

a domestic sewage: sewage coming from sewage coming from sewage coming from sewage coming from

industrial operations commercial establishments households offices

of excreta disposal includes the following, EXCEPT: bored hole latrine septic tank aqua privy pour flush latrine

82. The most practical and sanitary way to excrete disposal in the rural areas is: a. overhung latrine b. water-sealed toilet c. chemical toilet d. septic toilet 83.A gas produced by the biodegradation of waste is: a. Methane b. oxygen c. carbon monoxide d. carbon dioxide 84.In the most part of the world, vector control is largely through: a. genetic manipulation of vectors b. modifying the environment of vectors c. use of chemical pesticides d. extensive public health education 85.The probability that a person has the disease given a positive test result is: a. negative predictive value of the test b. sensitivity of the test c. specificity of the test d. positive predictive value of the test 86. A physician wants to determine the prevalence of parasitism among school children in an urban area. There is great variability in the economic status in that locality. On one hand, most children in private schools come from affluent families, and on the other hand, most children from public schools come from poor families. The most appropriate type of sampling methodology to use in this situation is: A. simple random sampling C. stratified random sampling B. systematic sampling D. multi-stage sampling 87. Philhealth benefits vary according to the following except: A. Relative unit value of surgery B. Actual cost of care C. Type of hospital D. Type of physician 88. One can be reasonably sure that the hypothesized cause preceded the occurrence of the disease and that disease status did not differentially influence the selection of subjects by study factor level in a: A. Cross-sectional study C. Case-control using prevalent cases B. Cohort study D. Experimental study design 89. The following method of data analysis is most relevant for the cohort study design: A. Prevalence odds ratio B. Prevalence rate C. Odds ratio D. Relative risk

52

90. This which is A. B. C. D.

is a sampling method wherein every subject falling within a fixed sampling interval is obtained from a population N arranged in some definite way: Simple random sampling Systematic sampling Stratified sampling Cluster sampling

91. When a researcher wants to test for the association between two categorical variables, the appropriate statistic to use is: a. Chi square b. T test c. Analysis of variance d. Correlation coefficient 93. The A. B. C. D.

rate that measures the probability of a person dying in a year regardless of age, sex, race, etc is: case fatality rate neonatal mortality rate crude death rate specific death rate

94. Which method of collecting data is devoid of observer error and bias? A. Interview B. Self-administered questionnaire C. Observation D. Review of documents 95. Used to etc. a. b. c. d.

describe the relative importance of different fatal disease in the population of different age, sex, occupation specific death rate attack rate cause of death rate proportionate mortality rate

96. The peculiar feature of a cross sectional study that distinguishes it from other study designs is: A. Information on both the independent and dependent variables are collected simultaneously at the time of the survey B. Information on the independent variable is collected first, after which the subjects are followed up later for assessment of the dependent variable C. Information on the dependent variable is obtained, after which assessment of the independent variable is made D. Information on both the independent and dependent variables are obtained by doing either queries or actual observation 97. The effect of an inadequate sample size for a study is: A. Subjects may not be a random representative of the base population B. There may not be enough subjects that will develop the outcome, in order to detect a difference C. The researcher will not be able to apply the appropriate statistical test for the kind of data available D. The researcher might find an association between independent and dependent variables, even if there actually is none 98. The group most vulnerable to nutritional deficiencies: a. Pregnant women b. Working group c. Adolescents d. Non-lactating mothers 99. The most sanitary source of water is: a. Groundwater b. Rainwater c. Surface water d. Brackish water 100. The primary disinfectant in water treatment: a. Fluorine b. Chlorine c. Aluminum sulfate d. Ferric sulfate SOURCES: Beaglehole, Bonita, Kjellstrom; BASIC EPIDEMIOLOGY STUDENT‟S TEXT, World Health Organization, July 1990

53

Clark, D., MacMahon, B: PREVENTIVE AND COMMUNITY MEDICINE, 2ND ed 1981 DOH Health Programs Manual Gupta, M.; Mahajan, B: TEXTBOOK OF PREVENTIVE AND SOCIAL MEDICINE, 3rd ed 2003 Jekel, Elmore & Katz; EPIDEMIOLOGY BIOSTATISTICS AND PREVENTIVE MEDICINE, 1996 Kuzma, Bohnenblust; BASIC STATISTICS FOR THE HEALTH SCIENCES 4th ed., McGraw-Hill 2001 Last, J; Wallace, R; PUBLIC HEALTH AND PREVENTIVE MEDICINE, 13th ed, 1992 Philhealth Manual Sanchez, Morelos, Baltazar, Peralta; RESEARCH METHODS IN HEALTH AND MEDICINE, Vol. 1 Planning Research (3rd ed.) Philippine Council for Health Research and Development 1996 Jmn/uermpcmexam questions for apmc may 4 2007 1.

This is a branch of medicine that deals with provision of comprehensive health care to people regardless of age or sex:

A. Public Health B. Community Medicine C. Preventive Medicine D. Family Medicine Answer: D –Proceedings of the Orientation Course in Family Medicine p. 8 3. This is the branch of medicine that deals with the study of the causes, distribution, and control of disease in populations: A. B. C. D. Answer: C – 4.

Family Medicine Preventive Medicine Epidemiology Public Health Epidemiology by Leon Gordis p. 2

Which of the A. B. C. D.

following is an example of chemoprophylaxis for newborns? Varicella vaccine Erythromycin ointment Folic acid BCG

Answer: B – Family Medicine Ambulatory Care and Prevention by Mengel et. al p. 736 5. Dr. Hook wants all of his children and their respective families to live with him and his wife until the time of his death. This is an example of what family structure? A. Nuclear B. Extended C. Single Parent D. Blended Answer: B – Proceedings of the orientation course in Family Medicine pp. 14-15 6. The stepfather and half-brother of Keanna lives with her and her mother. This is an example of what family structure? A. Nuclear B. Extended C. Single Parent D. Blended Answer: D - Proceedings of the orientation course in Family Medicine pp. 14-15 7. The family life cycle starts with this stage: A. Newly Married Couple B. Family With Young Children C. Unattached Young Adult D. Family With Adolescents Answer C: Proceedings of the orientation course in Family Medicine p. 18 8. In the family APGAR, resolve is elicited by A. asking if the family supports the patient in his desire for progress

54

B. looking into patient's desire to openly communicate problems with the family C. Asking if the patient feels loved by family members D. inquiring if the patient is given adequate time by the members of the family Answer D: Proceedings of the orientation course in Family Medicine p. 41 9. Which of the A. B. C. D.

following does not describe the CURRENT state of the Philippine health situation? Double burden of disease Continuing threat from emerging and resurgent diseases Highly centralized health care system Large variations in health status across population and socioeconomic groups

Answer C: Filipino Report Card on Health, Social Weather Stations Survey 2000 11. A score of 8 on the family APGAR indicates: A. A highly functional family B. A moderately dysfunctional family C. A severely dysfunctional family D. A large, extended family structure Answer A - : Proceedings of the orientation course in Family Medicine p. 41 13. Which of the following parameters are evaluated in SCREEM? A. Entrepreneurial B. Econom ic C. Resolve D. Mental Answer B - Proceedings of the orientation course in Family Medicine p. 48 14. A A. B. C. D.

dashed line (-------) that connect s a male and female couple in the family genogram denotes a married couple a legally separated couple a consanguinou s marriage live-in partners

Answer D - Proceedings of the orientation course in Family Medicine p. 46 16. You should use an 'arrow' symbol in your patient's genogram to signify: A. The index patient B. The head of the family C. Death in the family D. Provider of the family Answer A - Proceedings of the orientation course in Family Medicine p. 36 17. Stage in the illness trajectory experienced by the patient and family members prior to conta ct with medical care providers A. Onset of illness B. Impact Pha se C. Major therapeutic efforts D. Early adjust ment to outcomes Answer A - Proceedings of the orientation course in Family Medicine p. 29 18. The most catastrophic form of Cerebrovascular Accident A. Lacunar B. Thromoboembolic C. Hemorrhagic D. Transient ischemic attack Answer C – Harrison Principles of Internal Medicine 14 th ed. P. 2342 19. Chronic A. B. C. D.

bronchitis is a for at least 2 for at least 3 for at least 2 for at least 3

disorder months months months months

in which productive cough is present in most days of the year, for more than 3 consecutive years of the year, for more than 2 consecutive years of the year, for more than 2 consecutive years of the year, for more than 3 consecutive years

Answer B - Harrison Principles of Internal Medicine 14th ed p. 1451

55

20. “It measures what is supposed to be measured” refers to:

56

A. Specificity B. Sensitivity C. Validity D. Reliability Answer D. Epidemiology by Gordis p. 74 21. In Epidemiology, it is defined as the proportion of the true negatives correctly identified by the test. A. Specificity B. Sensitivity C. Validity D. Reliability Answer A. Epidemiology by Gordis p. 59 22. A. B. C. D.

It is one with potentially an infinite number of possible values in any interval. Continuous variable Bulbous variable Categorical variable Discrete variable

Answer A – Foundations of statistical analysis for the Health Sciences by Mendoza et. al p. 5 23. Another name for ranking scale, it has an implicit order relationship. A. Nominal B. Ordinal C. Interval D. Ratio Scales Answer B - Foundations of statistical analysis for the Health Sciences by Mendoza et. al p. 5 24. The A. B. C. D.

most frequently occurring value in a series of observations: Frequency distribution Median Weighted mean Mode

Answer D - Foundations of statistical analysis for the Health Sciences by Mendoza et. al p. 173 25. The A. B. C. D.

highest score minus the lowest score equals the: Percentile rank Interquartile difference Range Standard deviation

Answer C - Foundations of statistical analysis for the Health Sciences by Mendoza et. al p. 183 26. The A. B. C. D.

square root of variance: Standard deviation Interquartile range Percentile Kurtosis

Answer A - Foundations of statistical analysis for the Health Sciences by Mendoza et. al p. 188 27. This measure compares standard deviations between several investigations examining the same variable: A. Coefficient of variation B. Fractional integer C. Frequency distribution D. Quartile deviation Answer A - Foundations of statistical analysis for the Health Sciences by Mendoza et. al p. 189 28. All of the following are true of the normal distribution EXCEPT: A. It is symmetrical about the mean B. It is completely determined by 2 parameters: the mean and the standard deviation C. It has different values for the mean, median, and the mode D. It has long tapering tails that extend infinitely in either direction without touching the X axis Answer C – Foundations of statistical analysis for the Health Sciences by Mendoza et. al pp. 208-209 29. The 300 A. B. C. D.

weight of Filipino babies at birth was measured and found to have a mean of 2500 grams and a standard deviation of grams. Which of the following statements is true? 68.5% of the babies would have a weight ranging from 2200 to 2500 grams 95% of the babies would have a weight ranging from 1900 to 3100 grams 99.7% of Filipino babies would have a weight between 2200 and 2800 grams All of the babies measured will have a weight between 2200 and 2500 grams

Answer B - Foundations of statistical analysis for the Health Sciences by Mendoza et. al p. 211 30. Which of the following measures central of tendency will have the highest value in a distribution that is skewed to the left? A. Average B. Mean C. Median D. Mode

57

Answer D - Foundations of statistical analysis for the Health Sciences by Mendoza et. al p. 179 31.First step in a research process a. Introduction b. Research objective c. Hypothesis d. Conceptual definition e. Research problem Answer C- High-Yield Biostatistics by Lippincott Williams& Wilkins p.33 32.The frequency distribution of the sample mean that is generated by performing repeated sampling and obtaining the mean from each sample is called a. Sampling distribution b. Percentage tabulation c. Frequency tabulation d. None of the above Answer A- High-Yield Biostatistics by Lippincott Williams& Wilkins p.21 33.This refers to a statement about the value of a population parameter which is either rejected or not rejected in hypothesis testing a. Research question b. Null hypothesis c. Alternative hypothesis d. Statistical decision Answer B- High-Yield Biostatistics by Lippincott Williams& Wilkins p.33 34.The test statistic that is used to compare the proportions of three different groups is a. b. c. d.

z-test t-test Chi-square test Linear regression

Answer C- High-Yield Biostatistics by Lippincott Williams& Wilkins p.46 35.The process of determining a range of values such that the parameter of interest is included in this range is called a. Interval estimation b. Point estimation c. Hypothesis testing d. Critical region Answer A- High-Yield Biostatistics by Lippincott Williams& Wilkins p.53 37.In hypothesis testing, this refers to the probability of obtaining the observed result or a more extreme one under the null hypothesis a. b. c. d. Answer C- High-Yield

type I error power p-value type II error Biostatistics by Lippincott Williams& Wilkins p.37

38.In a study to compare the efficacy of two drugs, the statement “The proportion of patients that are cured by Drug A is the same as that by Drug B”, is called a a. b. c. d.

Null hypothesis Alternative hypothesis Statistical decision Research synthesis

Answer A- High-Yield Biostatistics by Lippincott Williams& Wilkins p.33 39.The first step in hypothesis testing is a. b. c. d. Answer D- High-Yield Biostatistics

State the level of significance Specify the test statistic State the alternative hypothesis State the null hypothesis by Lippincott Williams& Wilkins p.33

40.Which of the following statements is true about confidence intervals? a.The higher the confidence level, the wider the interval estimate. b.The higher the confidence level, the narrower the interval estimate. c.The confidence level does not affect the width of the confidence interval. d.None of the above Answer A- High-Yield Biostatistics by Lippincott Williams& Wilkins p.25

41.Incidence of a disease in a particular population over a specified time period: a.Morbidity rate C. Attack rate b.Mortality rate D. Crude rate

58

Answer A- High-Yield Biostatistics by Lippincott Williams& Wilkins p.72 42.The cumulative incidence of the disease when the duration of a disease is short and the observation period covers an entire epidemic: a. Case fatality rate C. Mortality rate b.Attack rate D. Morbidity rate Answer B- High-Yield Biostatistics by Lippincott Williams& Wilkins p.70 43..An individual who has contact with or who manifests the risk factor prior to becoming ill: a.Exposed C. Infected b.Inflicted D. Well Answer A- High-Yield Biostatistics by Lippincott Williams& Wilkins p.72 44.The infant mortality rate of a country in a given year is 45.1 per year per 1000 live birth means: a.45.1 infants died before reaching their first birthday b.45.1% of all deaths occurred in infants below one year c.45.1 infants died before reaching their first birthday per 1000 live births d.45.1 infants per 1000 of the population died Answer A- High-Yield Biostatistics by Lippincott Williams& Wilkins p.72 45.An extraneous variable is called a. Independent variable b.Confounding variable c.Dependent variable d.Exposure variable Answer B- High-Yield Biostatistics by Lippincott Williams& Wilkins p.60 46. . Performance of new born screening should be done : A. 24 hours after birth but not more than one week B. 24 hours after birth but not more than five days C. 24 hours after birth but not more than three days D. anytime after birth E. none of the above Ans. C- http://www.doh.gov.ph/health_laws.htm 47.The variable assumed to influence the problem a.Independent variable b.Confounding variable c,Dependent variable d.Continuous variable Answer B- High-Yield Biostatistics by Lippincott Williams& Wilkins p.58 48. Newborn Screening Act of 2004 is also known as: A. RA 9288 B. RA 9388 C. RA 9287 D. RA 9387 Ans. A- http://www.doh.gov.ph/health_laws.htm 49.. Penalties imposed in on individuals who violate the clean air act are the following except: A. first offense – one thousand pesos B. second offense – not less than two thousand pesos but not more than four thousand C. third offense – suspension of drivers license D. fourth offense – imprisonment of three months Ans. D- http://www.doh.gov.ph/health_laws.htm 50.The assumption of the relation between two variables to be studied a. Introduction b.Research objective c.Hypothesis d.Conceptual definition e. Research problem Answer C- High-Yield Biostatistics by Lippincott Williams& Wilkins p.33 51. An Act which was design to promote corneal transplantation in the country: A. Corneal Transplant ACT B. Organ Transplantation ACT of 1998 C. Enacted on February 20,1995 D. Enacted on May 20,1995 Ans. C- http://www.doh.gov.ph/health_laws.htm 52. A study that describes an interesting or unusual condition in a group of patients A. Case report B. Ecological study C. Cross-sectional study D. Case control study E. Case series Answer E- High-Yield Biostatistics by Lippincott Williams& Wilkins p.64 53.Recall bias is a common problem in what type of study

59

a. Case report b. Ecological study c. Cross-sectional study d. Case control study e. Cohort study Answer D- High-Yield Biostatistics by Lippincott Williams& Wilkins p.63 54. A study that establishes the association between risk factor and outcome variable a.Case report b.Ecological study c.Cross-sectional study d.Case control study e.Cohort study Answer E- High-Yield Biostatistics by Lippincott Williams& Wilkins p.62 55.In epidemiologic studies, the identification of causal factors is based on the experience of a.An individual b.A group of individuals c. Experimentation d. None of the above Answer B- High-Yield Biostatistics by Lippincott Williams& Wilkins p.68

56.This refers to statistical procedures that allow valid generalization of results from a sample to the population it is supposed to represent a.Statistical Association b.Causal Association c.Descriptive statistics d.Statistical inference Answer d- High-Yield Biostatistics by Lippincott Williams& Wilkins p.1 57.When one sets of the maximum probability of making an error of rejecting the null hypothesis when it is true, he is specifying the a.Critical region b.Statistical test c.Level of Significance d.Power Answer C- High-Yield Biostatistics by Lippincott Williams& Wilkins p.34 58.In statistical inference, this term refers to the total collection of elements of interest a.Population b.Sample c.Parameter d.Statistic Answer a Answer d- High-Yield Biostatistics by Lippincott Williams& Wilkins p.1- High-Yield Biostatistics by Lippincott Williams& Wilkins p.1 59.This refers to the procedures in assessing evidence from a sample, whether or not the data from the sample supports a hypothesis about a population a.Statistical inference b.Estimation c.Hypothesis testing d.Descriptive statistics Answer c- High-Yield Biostatistics by Lippincott Williams& Wilkins p.33 60.In a sample of 500 Grade I children examined, 450 were found to have dental caries. The proportion 90% computed by dividing 450 by 500 is an example of a.Population b.Sample c.Parameter d.Statistic e. unskilled workers Answer d- High-Yield Biostatistics by Lippincott Williams& Wilkins p.21 61.“Double burden of disease” means a. While the patient has to suffer the physiologic effects of an illness, he also has to contend with its psychological and economic impact b. While infectious diseases of poverty still abound, degenerative illnesses are on the rise. c. The top causes of morbidity always comes in a pair of related illnesses d. An infectious disease may be aggravated by a concomitant degenerative illness. Ans. B- Annual Report of the Field Health Service Information System pp 113-122 . Sec. Francisco Duque III 64.Which of the following does not describe the CURRENT state of the Philippine health situation? a.Double burden of disease b.Continuing threat from emerging and resurgent diseases c.Highly centralized health care system d.Large variations in health status across population and socioeconomic groups

60

Ans. C-

e.Health status generally improving but at a slower rate compared to neighboring Asian countries Annual Report of the Field Health Service Information System pp 113-122 . Sec. Francisco Duque III

65.In relation to the preceding question, which agency takes the principal responsibility for primary health care and service delivery at the community level as accorded by law? a.Local Government Chief Executives (like the Mayor, Governors) b.The Department of Health Secretary c.The President of the Republic d.The Doctors and midwives in the community Ans. D-

Annual Report of the Field Health Service Information System pp 113-122 . Sec. Francisco Duque III

66.After infancy, this is the most common cause of disease and deaths for young children, especially below 5 years of age a.Immunizable Infectious disease b.Degenerative diseases c.Emerging diseases like Avian Flu d.Malnutrition Ans. D-

Annual Report of the Field Health Service Information System pp 113-122 . Sec. Francisco Duque III

67. RA 7883 is also known as: A. Medical workers benefits and compensation act B. Barangay Health Workers Incentives and Benefits Act of 1995 C. Barangay Health Workers Benefits Act of 2003 D. Voluntary Blood Donation Act of 2002 Ans. B- http://www.doh.gov.ph/health_laws.htm 68. Hospital Licensure Act, which requires that all hospitals be licensed is also known as: A. RA 4226 B. RA 2426 C. RA 2264 D. RA 1235 Ans. A- http://www.doh.gov.ph/health_laws.htm 69. The National Blood Services Act was enacted into law in: A. 1996 B. 1995 C. 2000 D. 1994 Ans. D- http://www.doh.gov.ph/health_laws.htm 70. An Act establishing Philhealth is also known as: A. Philippine Health Insurance Act of 1995 B. National Health Insurance Act of 1995 C. National blood donation Act of 1995 D. Philippine Health Insurance and Benefits Act of 1995 Ans. D- http://www.doh.gov.ph/health_laws.htm 71. Traditional and Alternative medicine act of 1997 established this to promote traditional and alternative medicine. A. TAMATH B. PATIHC C. PITAHC D. TAMHC Ans. C- http://www.doh.gov.ph/health_laws.htm 72. TAMA was established in: A. June 28, 1997 B. August 28,1996 C. September 30,1998 D. July 28, 1997 Ans. D- http://www.doh.gov.ph/health_laws.htm 73. Republic Act 8344 is an amendment to : A. PB 107 B. PB 239 C. PB 701 D. PB 578 Ans. B- http://www.doh.gov.ph/health_laws.htm 73. On of the policies of this Act is to protect the consumer from deceptive, unfair and unconscionable sales act and practices. A.RA 7393 B. Consumer Act of the Philippines C. Both D. None of the Above Ans. B- http://www.doh.gov.ph/health_laws.htm 74.The level of state involvement is lowest in which policy instrument? a.Auction of property rights b.Information and exhortation c.Subsidy d.Regulations 75. An Act regulating the packaging, use, sale and distribution of tobacco and other related products. A.RA 9211 B. PB 9211 C. tobacco protection act of 2003

61

D. Tobacco Distribution Act of 2002 Ans. A- http://www.doh.gov.ph/health_laws.htm 77. The Magna Carta of Public Health Workers, also known as RA 7305 was enacted on: A. April 24,1993 B. March 26, 1992 C. April 24, 1994 D. May 1,1993 Ans. B- http://www.doh.gov.ph/health_laws.htm 78. To promote and encourage voluntary blood donation is one of the policies of the: A. Blood Donation Act B. Organ Donation Act C. Blood Services Act D. Organ Services Act Ans. C- http://www.doh.gov.ph/health_laws.htm 79. An Act regulating the practice of pharmacy in the Philippines: A. RA 5678 B. RA 5932 C. RA 5921 D. RA 5934 Ans. C- http://www.doh.gov.ph/health_laws.htm 80. RA 8344 is also known as: A. National AIDS Prevention Act B. Tobacco Distribution Act C. Act Preventing Hospital Deposits D. Clean Air Act Ans. C - http://www.doh.gov.ph/health_laws.htm 81. Conducting an ocular view of all existing hospitals in the Philippines is one of the duties of the : A. DPWH B. Department of Sanitation C. DOH- Hospital Services D. Hospital Licensure Board Ans. C- http://www.doh.gov.ph/health_laws.htm 82. An act to promote, require and ensure the adequate supply, distribution, use and acceptance of drugs and medicines identifies by their generic names. A. Generics Act B. Pharmacy Regulation Act C. AIDS Act D. RA 6665 Ans. A- http://www.doh.gov.ph/health_laws.htm 83. An Act prohibiting the sale, distribution and manufacture of counterfeit drugs: A. RA 8302 B. RA 8023 C. RA 8203 D. RA 8032 Ans. A- http://www.doh.gov.ph/health_laws.htm 84. The Clean Air Act of 1999 was enacted on: A. June 23, 1998 B. July 28,1999 C. July 27,1999 D. July 27,1998 Ans. D- http://www.doh.gov.ph/health_laws.htm 86. Included in the mandate of this act is the reporting of HIV/ AIDS positive individuals for the purpose of monitoring AIDS cases in the Philippines: A. HIV monitoring and control act of 1997 B. Philippine AIDS Prevention and Control Act of 1998 C. Philippine AIDS Control Act of 1998 D. AIDS ACT of 1997 Ans. B- http://www.doh.gov.ph/health_laws.htm 87.When was the Alma Ata declaration initially intended to be achieved? a. Year 1990 b. Year 2000 c. Year 2015 d. Undetermined Ans B -Health for all beyond 2000: the demise of the Alma-Ata Declaration and Primary Health Care in developing countries; John Hall and Richard Taylor entitled " 88.When are the Millennium Development Goals (MDGs) initially intended to be achieved? a.Year 1990 b.Year 2000 c.Year 2015 d.Undetermined Ans. C-HTTP://www.un.org/millenniumgoals 89.Which of the following is not a direct health-related MDGs? a.Reduce child mortality by two-thirds among children under five b.Improve maternal health by reducing maternal mortality by three-quarters c.Combat HIV/AIDS, Malaria, and other diseases

62

d.Achieve universal primary education Ans. d-HTTP://www.un.org/millenniumgoals 90.Which of the following is not an MDG? a.Achieve universal primary education b.Promote gender equality and empower women c.Combat HIV/AIDS, Malaria, and other diseases d.Achieve Universal health promotion activities Ans. D-HTTP://www.un.org/millenniumgoals 91.Which approach enumerated below emphasizes the need for community participation in achieving betterment for health? a.Primary Care Approach b.Millennium Development Goals c.Primary Health Care approach d.Eradication of Extreme Poverty and Hunger 92.Which approach enumerated below emphasizes the need for investments in health to achieve national development? a.Primary Care Approach b.Millennium Development Goals c.Primary Health Care approach d.Eradication of Extreme Poverty and Hunger Ans. B-HTTP://www.un.org/millenniumgoals 93.Jeffrey Sachs, a Harvard-educated economist, authored the document commissioned by the World Health Organization (WHO) that eventually established the global funds. The said funds pooled monetary and various resources together at the global level and earmarked these into Programs and activities targeted to eradicate debilitating diseases, especially in aid of developing countries. What is the title of his document? a.WHO Commission on Macroeconomics and Health b.WHO Commission on Health-related Economics c.Primary Health Care d.The Commission on the Millennium Development Goals Ans. A.-MACROECONOMICS AND HEALTH: INVESTING IN HEALTH FOR ECONOMIC DEVELOPMENT by Jeffrey Sachs pp 1-17 Executive Summary 94.The MDGs stipulated reductions in maternal and child mortalities by a fraction of a benchmark value. The MDGs are compared to the situation during which year that served as benchmark? a.1990 b.2000 c.2015 d.undetermined

63

Ans. A.-HTTP://www.un.org/millenniumgoals 95.Which of the following statements is false? a.The document that served as basis for the MDGs showed evidences between the links of disease and poverty. b.The Alma Ata Declaration is obsolete since it should have been achieved in the year 2000. c.The Alma Ata Declaration uses the primary health care approach. d.The Millennium Development Goals is conceptualized behind the idea that better health means better productivity and vice-versa. e.The MDGs push us to achieve better investments for health. Ans B -Health for all beyond 2000: the demise of the Alma-Ata Declaration and Primary Health Care in developing countries; John Hall and Richard Taylor entitled " 96.Which of the following is not an essential element of primary health care? a.Immunization b.Essential Medicines c.Maternal Mortality d.Water and Sanitation Ans C -Health for all beyond 2000: the demise of the Alma-Ata Declaration and Primary Health Care in developing countries; John Hall and Richard Taylor entitled " 97.Which of the following is not an essential element of primary health care? a.Nutrition and supplemental feeding b.Essential Medicines c.Maternal and Child Health d.Population Control and Management Ans D -Health for all beyond 2000: the demise of the Alma-Ata Declaration and Primary Health Care in developing countries; John Hall and Richard Taylor entitled " 98.Which of the following is not an essential element of primary health care? a.Immunization b.Endemic Diseases c.Universal Primary Education d.Curative Care Ans C -Health for all beyond 2000: the demise of the Alma-Ata Declaration and Primary Health Care in developing countries; John Hall and Richard Taylor entitled " 99.What year was the Alma Ata Declaration adopted by over 130 WHO Member-States, including the Philippines? a.1958 b.1968 c.1978 d.1988 e.1998 Ans C -Health for all beyond 2000: the demise of the Alma-Ata Declaration and Primary Health Care in developing countries; John Hall and Richard Taylor entitled " 100.According to the WHO, health is defined as: a..Not merely the absence of disease b.The absence of infirmity c.The complete physical, mental and social well-being of a person d.All of the above e.None of the above Ans D-Health for all beyond 2000: the demise of the Alma-Ata Declaration and Primary Health Care in developing countries; John Hall and Richard Taylor entitled " 1.

In what A. B. C. D.

case should a family case conference or meeting be of least help? child admitted for LBM and vomiting of three days duration Two year old newly diagnosed to have thalassemia, a blood disorder Fifty three year old working mother diagnosed with Stage III breast CA Forty five year old father with TB who remains symtomatic on the 5 th month of medication

2.

The single most difficult time in the entire illness experience is A. onset of illness B. diagnosis of major debilitating or terminal illness C. financial aspects of application of therapy D. adjustment to permanency of outcome

3.

Total evaluation of health care involving family relations, social and cultural systems leading to better evaluation of health care A. family life cycle B. family systems theory C. family set-up D. family relationships/parent-child interaction

4.

DOH program to control diarrhea is targeted for A. under 5 years of age B. children and pregnant women

64

C. D.

lactating mothers 2-14 years old

5.

The following are covered by Philhealth except A. legitimate married 19 year old daughter B. thirty year old legally blind man C. sixty five year old mother working at a sari-sari store but depends for most needs on employed forty year old son D. spouse who is not an NHIP member

6.

In 1992 all administrative powers were transferred to political units and health managers came under the management of non-health managers. A. centralization B. devolution C. reorganization D. restructuring

7.

One of the guiding principles of RA 7875 is universality which refers to A. adequately meeting the needs for personal health services at various stages of a member‟s life B. provision to all citizens the mechanism of gaining financial access to health services C. provision of uniform basic benefits D. balancing economical use of resources with quality of care

8.

Membership in the NHIP shall take effect A. three days after payment of premiums B. upon enrollment and payment of premiums C. at the time the member gets sick D. one month after enrollment and payment of premiums

9. One of the following characterizes the “essential drug” concept A. should be implemented by those in the government sector B. one could select about 250 essential drugs for 90% of the diseases in the country C. does not allow patients to choose from a list of generic drugs D. discrepancy between availability of drugs and needs of population 10. The following are categories of health services granted to the member or his dependents for in-patient hospital care A. room and board B. fee for privately hired nurse C. personal preventive services D. food supplements 11. Ability of the body to produce antitoxin with diphtheria vaccine A. agent characteristic B. host characteristic C. host-agent characteristic D. agent-environment characteristic 13. Referral A. B. C. D.

to home care program of a diagnosed cancer patient primary prevention secondary prevention tertiary prevention primordial prevention

14. The amount of chopped fresh leaves of lagundi for adults is A. two teaspoons B. two tablespoons C. four tablespoons D. six tablespoons 15. One of the following is used as diuretic and anti-urolithiasis agent A. ulasimang bato B. sambong C. yerba Buena D. guava 16. Ampalaya leaves is recommended for A. insulin-dependent diabetes mellitus B. non-insulin dependent diabetes mellitus C. non-toxic goiter D. toxic goiter 17. One of the following medicinal plants contains vinca alkaloids and may be used as an anticancer agent A. adelfa B. aloe vera C. rosas de baybayon D. tawa-tawa 18. When garlic is used to lower cholesterol, it should not be A. broiled B. soaked in vinegar C. fried D. raw 19. Stability of vitamins under refrigeration

65

A. B. C. D.

agent-environment interaction host-environment interaction human host-agent interaction agent-host-environment interaction

20. Second line of defense against disease agents A. secretions B. respiratory cilia C. mucous membranes and nails D. lymph glands 21. Per single period of confinement a general practitioner will be paid by the Philhealth A. B. C. D.

500 150 300 200

pesos/day pesos/day pesos/day pesos/day

22. Strategies to control TB in the Philippines except A. B. C. D.

DOTS for all patients Free medicines available for patients in public hospitals only Active participation of private practitioners in NTP NTP reaches out to physicians in private sector

23. Essential elements of primary health care except A. B. C. D.

MCH including EPI Basic sanitation and safe water Provision of food supply Essential drugs available

24.One of the advantages of decentralizing the health system is A. Greater representation of various political, religious, ethnic, and tribal groups in development decision-making. B. C. D.

Bureaucratic and highly structured nature of the health system facilitates more efficient delivery of services Lesser participation of citizens in development planning and management, hence, lesser conflict Greater control of officials in the central office as regards to planning and implementation of programs

25. According to Rule IX of the Magna Carta, a public health worker who is regularly employed under permanent status shall have security of tenure A. But he can be dismissed anytime from work if his boss finds him unsatisfactory for the job B. If he is unjustly dismissed from the service, he/she shall be entitled to reinstatement without loss of seniority rights and payment of back wages/salaries and other benefits C. And can never be terminated from his job D. But is not entitled to benefits and back wages if unjustly dismissed from service 26. Family type that includes stepparents and stepchildren brought about by annulment, separation and remarriage A. B. C. D.

nuclear extended blended communal

27. True of systems concepts of family by Minuchin except A. adapting to change is hallmark of healthy functioning B. symptoms of our member have effect within the family C. repeating interaction patterns regulate behavior of members D. usually there are victims and victimizers in the family 28. Family structure where relationship exists between at least three people and two collude against the other A. B. C. D.

coalition alliance role selection boundaries

29. Enmeshment as family process occurs in the following A. B. C. D.

mother-child subsystem excludes father in parenting third person comes into a two-pair system to diffuse conflict members are overly reactive to stress on one member parents have privacy from children

30. True of family genogram construction except A. B. C. D.

consist of at least three generations firstborn is farthest to the right with sibling following to the left horizontal lines connects children with spouses children connected to parents with vertical lines

66

31. In APGAR, this measures the satisfaction attained in solving problems by the communicating A. B. C. D.

Affection Resolve Adaptation Partnership

32. A mother who insists to accompany her son enrolling in medicine proper during the routine physical exam in school A. Triangulation B. Coalition C. Disengagement D. Enmeshment

33. A daughter sides with her mother regarding their problem with the father. They accuse the father having another woman A. B. C. D.

coalition disengagement triangulation enmeshment

34. A family with three children ages 9, 10, and 15 living with their grandparents in the city have adopted a 2-year old boy. The family‟s APGAR is 3. This family‟s function is A. B. C. D.

normal mildly dysfunctional moderately dysfunctional severely dysfunctional

35. Atherosclerotic changes in coronary vessels A. Stage of Susceptibility B. Stage of Presymptomatic disease C. Stage of Clinical disease D. Stage of Disability 36. Measles A. B. C. D.

followed by the development of subacute sclerosing panencephalitis Stage of Clinical disease Stage of Susceptibility Stage of Presymptomatic disease Stage of Disability

37. Secondary prevention except A. specific protection B. disability limitation C. prompt treatment D.

early diagnosis

38. Health promotion except A. use of immunizations B. genetics C. personality development D. periodic selective examination 39. Levels of preventive measures affecting reaction of the host to the stimulus except A. B. C. D. 40. Tertiary A. B. C. D.

specific protection rehabilitation disability limitation early diagnosis and prompt treatment prevention except sheltered workshop half-way homes occupational therapy improvement in working conditions in dusty industries

41. Early diagnosis and prompt treatment in cancer include the following except A. Recruitment and training of specialists B. Self-examination of breast C. Cancer detection centers D. Selective examination 42. Ability to stimulate the host to produce agglutinins, opsonins, antitoxins, etc. A. Pathogenicity B. Virulence C. Antigenicity D. Infectivity 43.Measure A. B. C.

of ability of an agent when lodged in the body to set up local or general tissue reaction Pathogenicity Virulence Infectivity

67

D.

incubation period

ondition which began the train of events which led to death A. immediate cause of death B. intervening cause of death C. intermedial cause of death D. underlying antecedent cause 45. Which of the following is true regarding physiologic adjustments in temperature? A. The percent of the body surface wetted with sweat decreases with the need to lose heat until maximum sweating occurs. B. The sweat, which runs off, the body is of great value in removing heat from the body, since heat is lost only when sweat is evaporated. C. If the temperature and humidity are both high, the body may be unable to lose sufficient heat. D. Men do not voluntarily replace the water lost by sweating; the sensation thirst is quite adequate. 46. Relation A. B. C. D.

of high temperature and humidity to physical work Muscular work contributes little to the total body heat load than environmental conditions Rectal temperature is most closely related to the rate of physical work The capacity of a man to work drops off at high temperature because of lack of oxygen supply. A marked increase in lactic acid in the blood limits the capacity of man to work at high temperature.

47. If the body is immersed in cold water, rather than cold air A. body temperature falls gradually B. air conducts heat faster than water C. air has a higher specific heat value D. entire body surface loses more heat to the water 48. This is the 2nd most abundant greenhouse gas: A. nitrous oxide B. CFC‟s C. Carbon dioxide D. Methane 49. This is not necessary in the formation of Ozone A. hydrocarbons B. SO2 C. Nitrous oxide D. Sunlight 50. The process of removing or destroying all microbial life of an object A. antisepsis B. C. D.

sterilization disinfection antimicrobial

51. Exert their action by injuring plasma membranes A. phenols B. biguanides C. halogens D. alcohols 52. True on A. B. C. D.

halogens as disinfectants except Iodine may combine with certain amino acids to inactivate enzymes Germicidal action of chlorine is based on the formation of hyperchlorous acid when chlorine is added to water Iodine is available as a tincture (combined with an organic molecule). Chlorine is used as a disinfectant in gaseous form

53. The following statements are correct regarding surface-active agents except A. Acid anionic detergents are used to clean dairy equipment B. Soaps assists in the removal of microorganisms through scrubbing C. Detergents decrease the surface tension among molecules of a liquid D. Soaps have a wide range of germicidal action 54. True concerning radiation except: A. Microwave can kill microbes indirectly as materials get hot. B. The effects of radiation depend on wavelength and duration C. Ultraviolet radiation has a high degree of penetration and exerts primary by ionizing water D. X-rays exert its effect by forming highly reactive hydroxyl radicals 55. The seven target diseases are: A. measles, polio, pneumonia, tetanus, chicken pox, TB B. diphtheria, mumps, polio, Tb, tetanus, measles C. TB, polio, measles, diphtheria, pertussis, tetanus, hep B D. Tetanus, pertussis, tonsillitis, measles, tetanus, TB 56. Extremely low frequency electromagnetic fields can cause brain tumor and this is emitted by A. B. C. D.

television sets laser printers Xerox machines Microwave oven

68

The most relevant part of the death certificate because it is the one counted in the statistics since it is the basic c 57. A 20-year-old female in respiratory distress was brought to the emergency room for intentional ingestion of a poisonous substance. Of the following, which is the most appropriate thing to do first. A. B. C. D.

Get a detailed history Maintain vital signs Give antidote Induce vomiting

58. Contamination of food by cockroaches is as example of : A. vehicle-borne transmission B. air-borne transmission C. mechanical transmission D. biological transmission 59. Gas responsible for explosion in garbage dumps: A. carbonic acid gas B. carbon monoxide C. methane D. nitrogen 60. Usual speech frequency is: A. 0 db B. 120 db C. 500 to 2000 Hz D. 20 to 20,000 Hz 61. Most likely to occur during cases of bomb explosions: A. conductive hearing loss B. noise –induced deafness C. sensorineural hearing loss D. temporary threshold shift 62. In storing vaccines, the following should be remembered A. DPT vaccine can be stored in the freezer and thawed B. The diluent should be in the area where the solvent is C. Never keep vaccine in the door of the refrigerator D. If the vaccine stored above or below safe temperature it will not lose its potency if it is for a short period of time only. 63. In transporting vaccine using a vaccine carrier this vaccine should be placed in contact with frozen ice packs A. BCG B. OPV C. DPT D. TT This vaccine A. B. C. D.

is most easily damaged by sunlight polio vaccine BCG DPT Tetanus toxoid

65. A fully immunized child has finished A. BCG, 2 DPT, 2 OPV B. BCG, 3 DPT, 3 OPV C. BCG, 3DPT, 3 OPV, Measles D. 3 DPT, 3 OPV, Measles 66. A 21 year old woman is pregnant with her first child, she had had received 2 doses of DPT when she was a child. How many doses does she need so that all infants born to her will be protected? A. 2 B. 3 C. 4 D. 5 67. Normal reaction to BCG vaccination are the following except A. small and tender swelling about 10mm across after 2 weeks B. swelling became small abscess when vaccinated after 2-3 weeks C. formation of scar after about 10 weeks D. acute inflammation reaction appearing 4-2 days of vaccination 68. The separation of infected persons for the period of communicability to prevent transmission of infectious agent to those who are susceptible is A. quarantine B. surveillance C. isolation D. herd immunity 44. 69. The prevention transmission of highly contagious or virulent infection that may be spread by both air and contact is A. strict isolation B. contact isolation

69

C. D.

respiratory isolation drainage precaution

70. These diseases are internationally quarantinable diseases and thus are labeled as Diseases Under Surveillance by WHO. A. B. C. D.

plague, cholera, smallpox plague, typhoid fever, cholera leprosy, smallpox, plague malaria, leprosy, smallpox

72. When shellfish is harvested from a contaminated area, it should purge before consumption for approximately: A. 1 day B. 1 week C. 2 weeks D. 2 days 73. One of the neurotoxins found in coal: A. cadmium B. chromium C. manganese D. arsenic 74. The most important test for potability of water. A. taste and odor B. chemicals C. algae, microorganism D. strep coliforms 75. Examples of water-borne diseases EXCEPT : A. amoebiasis B. minimata disease C. poliomyelitis D. schistosomiasis 76. Man is the definitive host in: A. dengue fever B. filiariasis C. malaria D. Rocky mountain spotted fever 77. Rats and other rodents are the reservoir and source of several diseases of man except A. B. C. D.

schistosomiasis salmonellosis dengue hemorrhagic fever plague

78. Which should be first priority in disaster relief? A. Safety of food supply B. Vector control C. Excreta disposal D. Promotion of personal hygiene 79. Early effects of ionizing radiation includes the following except A. anorexia, nausea, vomiting B. marrow syndrome C. sterility D. increase childhood cancer 80. The following are environmental sources of occupational stress except A. excessive noise B. attitudinal problems C. hot/cold fever D. posture 81. The bacterial limits for pasteurized milk and milk products is: A. 100, 000/ml B. 10,000/ml C. 20,000/ml D. 1gm/ml 82. A selective, partial limitation of freedom of movement of contacts, on the basis of known or presumed differences in susceptibility and related to danger of disease transmission. A. absolute quarantine B. modified quarantine C. isolation D. surveillance 84. What type of sampling technique is used for “Man-in-the-street” surveys and interviews among senior citizens who drop in at a health booth in SM City?

70

A. B. C. D.

simple random systematic haphazard cluster

85. What is the ability of a study to demonstrate an association if one exists? A. alpha B. P value C. power D. Type II error 86. In contrast to observational studies, experimental research: A. attempts to describe prevailing trends occurring in a population B. empirically tests a cause-effect relationship among variables C. requires two or more population groups in a study D. identifies risk factors accompanying the independent variable 87. Any of the following may be included as an “exposure” in health research except: A. potential risk factor B. The outcome being studied C. The treatment or therapy D. A health problem 88. The following are impact programs of DOH except A. soil-transmitted helminthes disease control B. rabies control C. malaria control D. environmental health services 89. Type of outbreak where exposure is brief and all cases develop within one incubation period of the disease. A. common source outbreak with intermittent exposure B. common source outbreak with continuous exposure C. point source outbreak D. propagated outbreak 90. A distribution that is negatively skewed hasA. higher frequency on the right B. higher frequency on the left C. a flatter distribution than the normal distribution D. a more peaked distribution than the normal distribution 91. Which has a wider interval? A. 80% confidence interval B. 90% confidence interval C. 95% confidence interval D. 99% confidence interval 92. Aims to reveal the existing gaps in knowledge regarding the problem being studied A. B. C. D.

significance of the study research question review of literature conceptual framework

93. A nominal scale: A. is made up of labeled or named categories with no implied order B. is a ranking scale C. has an ordering of values with one better than the other D. has equal distances between values 94. In a stratified sampling: A. k is determined by dividing the number of items in the sampling frame by the desired sample size B. the population is first divided into relevant subgroups and a random sample is selected from each C. the probability that a subject may be selected is unknown D. the assignment of subjects to treatments is done by using random methods 95. Which is A. B. C. D.

used to test for differences among proportions? paired t-test B. 2-sample t-test ANOVA D. Chi-square test

96. An investigator conducts a historical cohort study to explore the relationship between perimenopausal exogenous estrogen use and the risk of coronary heart disease (CHD). A total of 5000 exposed and 5000 unexposed women are enrolled and followed for 15 years for the development of myocardial infarction (MI). A total of 200 estrogen users and 300 nonusers had MIs. What is the relative risk for MI? A. B. C. D.

0.33 0.50 0.67 1.5

97. A study was undertaken to compare results of the surgical treatment of duodenal ulcer. A total of 1,358 patients who me t the study criteria were randomly assigned to one of four surgical procedures. The purpose of the randomization was to -

71

A. B. C. D.

ensure that the double-blind aspect of the study was maintained obtain the unbiased distribution of good- and poor-risk patients in all treatment groups achieve the same number of patients on each operation guarantee that the study group was a representative sample of the general population

98. A newly developed test for diabetes produced positive results in 138 of 150 known diabetics and in 24 of 150 persons known not to have diabetes. What is the sensitivity of the test? A. 84% B. 85.2% C. 88% D. 92% 99.

Treatment of a patient with gonorrhoea constitute: A. primary prevention for the patient, secondary prevention for potential contact B. secondary prevention for patient, primary prevention for potential contact C. secondary prevention for patient, tertiary prevention for contact D. tertiary prevention for contact, primary prevention for patient

100. An indirect health impact of climate change: A. heat wave in Europe in 2003 causing more than 20,000 deaths B. respiratory illness due to an increase in ozone C. altered concentration of fungal spores causing respiratory problems D. El Niño events causing dengue outbreaks

72

View more...

Comments

Copyright ©2017 KUPDF Inc.
SUPPORT KUPDF